Ophthalmology Flashcards

1
Q

Describe the borders of the orbital cavity

A
  • Pyramidal structure, apex points posteriorly, base anteriorly
  • Four bony walls
    • Medial wall- ethmoid, maxilla, lacrimal, lesser wing sphenoid (MLES)
    • Lateral wall- zygomatic bone, greater wing sphenoid (2)
    • Superior wall- frontal bone, lesser wing sphenoid (2)
    • Inferior wall- maxilla, palatine, zygomatic bone (3)
  • 7 bones: Many Friendly Zebras Enjoy Lazy Summer Picnic
    • Maxilla, Frontal bone, Zygomatic bone, Ethmoid bone, Lacrimal bone, Sphenoid bone, Palatine bone
How well did you know this?
1
Not at all
2
3
4
5
Perfectly
2
Q

There are three main pathways into the orbit at the orbital apex; what are they and what do they transmit?

A
  1. Optic canal
    1. Optic nerve
    2. Ophthalmic artery
  2. Superior orbital fissure
    1. Oculomotor nerve
    2. Trochlear nerve
    3. Abducens nerve
    4. Ophthalmic nerve (Va)
    5. Superior ophthalmic vein
  3. Inferior orbital fissure
    1. Infraorbital nerve (branch of maxillary CN Vb)
    2. Inferior ophthalmic vein
    3. Sympathetic nerves
How well did you know this?
1
Not at all
2
3
4
5
Perfectly
3
Q

What is the main arterial supply of the orbit?

A
  • The eyeball receives arterial blood primarily from ophthalmic artery
    • This is a branch of the internal carotid artery, arising immediately distal to the cavernous sinus
  • Branches of ophthalmic artery-
    • Central retinal artery supplying internal surface of retina
      • Occlusion à blindness
  • Venous drainage
    • Superior ophthalmic vein
    • Inferior ophthalmic veins
    • These drain into cavernous sinus
How well did you know this?
1
Not at all
2
3
4
5
Perfectly
4
Q

Describe the layers of the eyelid

A
  • Skin and subcutaneous tissue (most superficial)
  • Orbicularis oculi
    • Palpebral part- gentle closing of eyelids
    • Lacrimal part- tear drainage
    • Orbital part- tight closing of eyelids
    • Innervated by zygomatic & temporal branches of facial nerve
  • Orbital septum
    • Tough thin sheet of fibrous tissue originating from orbital rim periosteum blends with tarsal plates
    • Acts as a barrier against superficial infection spreading from the pre-septal space (subcutaneous tissue, orbicularis oculi) and post-septal space (orbital cavity proper)
  • Tarsal plates
    • Dense CT scaffolding
      • Provide stability & convexity to the lids
    • Superior tarsus = levator palpebrae superioris attachment
    • Inferior tarsus
    • Contain meibomian glands (tarsal glands)- secrete oily substance that slows evaporation of the eye’s tear film & prevents eyelids sticking together when closed
  • Levator apparatus
    • Levator palpebrae superioris
      • Opens the eyelid
      • Innervation- superior branch of oculomotor nerve
    • Superior tarsal muscle
      • Assists LPS in opening eyelids
      • Innervation- sympathetic fibres
  • Conjunctiva
    • Thin mucous membrane reflected on sclera of eyeball
    • Includes goblet cells that produce the mucous layer of the tear film
How well did you know this?
1
Not at all
2
3
4
5
Perfectly
5
Q

Describe the difference between a stye and a chalazion?

A
  • A stye is an infection of a hair follicle or Meibomian glands around the eyelash, painful and self-limiting
  • A chalazion is a painless granuloma of the Meibomian glands, absence of pain
How well did you know this?
1
Not at all
2
3
4
5
Perfectly
6
Q

Describe how compartment syndrome in the orbit can occur?

A
  • Uncommon surgical emergency
  • Acute rise in orbital pressure
  • Causes- retrobulbar haemorrhage from trauma
  • Haemorrhage into orbit, compression of ophthalmic artery, ischaemia of optic nerve, fixed dilated pupil, severe pain, ischaemia of ocular muscles too
How well did you know this?
1
Not at all
2
3
4
5
Perfectly
7
Q

Describe the contents of the orbital cavity?

A
  • Eyeball
  • Fat
  • Associated extra-ocular muscles
  • Nerves and blood vessels
  • Lacrimal apparatus
How well did you know this?
1
Not at all
2
3
4
5
Perfectly
8
Q

What is the lacrimal apparatus?

A
  • Lacrimal gland- anteriorly in the superolateral aspect of orbit, within the lacrimal fossa (depression in orbital plate of frontal bone). Produces watery serous liquid- lacrimal fluid
  • After secretion, lacrimal fluid circulates across the eye and accumulates in the lacrimal lake
  • Then drains into lacrimal sac (dilated end of the nasolacrimal duct)
  • Lacrimal duct- fluid empties into the inferior meatus of the nasal cavity
How well did you know this?
1
Not at all
2
3
4
5
Perfectly
9
Q

Briefly describe the action and innervation of the extraocular muscles?

A
  • Levator palpebrae superioris
    • Elevates upper eyelid
    • Oculomotor nerve
    • Superior tarsal muscle within- sympathetic innervation
  • Superior rectus
    • Elevates + adducts + medial rotation
    • Oculomotor nerve
  • Inferior rectus
    • Depresses + adducts + lateral rotation
    • Oculomotor nerve
  • Medial rectus
    • Adducts
    • Oculomotor nerve
  • Lateral rectus
    • Abducts
    • Abducens nerve
  • Superior oblique
    • Depresses + abducts + medial rotation
    • Trochlear nerve
  • Inferior oblique
    • Elevates + abducts + lateral rotation
    • Oculomotor nerve
How well did you know this?
1
Not at all
2
3
4
5
Perfectly
10
Q

What are the 3 layers of the eyeball?

A
  1. Outer: fibrous layer-
    1. Sclera- attachment for extraocular muscles, optic nerve penetrates through. Visible as the white part of the eye
    2. Cornea transparent & continuous with the sclera at the front, refracts light
  2. Middle: vascular layer- 3 continuous parts
    1. Choroid- ct and blood vessels, nourishes outer layers of retina
    2. Ciliary body- muscle and processes- control shape of lens & contributes to formation of aq. humour
      The ciliary processes project from surface of ciliary body, attach the lens to the ciliary body- controls shape of lens
    3. Iris- gives eye colour, aperture in centre called the pupil
  3. Inner: retina
    1. Pigmented outer layer- single layer of cells, absorbs light, prevents scattering of light within the eyeball
    2. Neural inner layer- consists of photoreceptors
    3. Centre of retina = macula- highly pigmented-
    4. Fovea- highest concentration of light detecting cells, only cones, high acuity vision
How well did you know this?
1
Not at all
2
3
4
5
Perfectly
11
Q

Pituitary macroadenoma- how does it present clinically, what imaging is best, and what is the treatment?

A
  • Sellar mass
  • Pituitary gland not seen separately
  • Suprasellar extension- optic chiasm
  • Rarely parasellar
  • Clinically: bitemporal hemianopia
  • Imaging: MRI best
  • Surgery: transsphenoidal
How well did you know this?
1
Not at all
2
3
4
5
Perfectly
12
Q

Mimics of a PCA infarction? Presenting complaint: homonymous hemianopia

A
  • Abscess- acutely unwell
  • Metastasis- hx of cancer
How well did you know this?
1
Not at all
2
3
4
5
Perfectly
13
Q

Describe the visual pathway

A
  • Photons of light enter the eye, stimulating the photoreceptors (rods and cones) in the retina
  • The photoreceptors synapse with retinal bipolar cells which transmit these signals to retinal ganglion cells
  • The retinal ganglion cells converge at the optic disc, forming the optic nerve
  • The optic nerve exits the eye, travelling through a defect of the lamina cribrosa of the sclera
  • The optic nerve can be considered an extension of the forebrain as it is covered by the meninges of the CNS
  • The optic nerve travels through the bony orbit and enters the middle cranial fossa through the optic canal (defect in lesser wing of sphenoid)
  • The optic nerve then travels along the floor of the middle cranial fossa, through the medial aspect of the cavernous sinus
  • Left and right optic nerves converge at the optic chiasm, which is located directly above the sella turcica of the sphenoid bone
  • Fibres from the nasal aspect of each retina decussate at the chiasm, whilst fibres from the temporal retina remain on their respective sides
  • The optic tracts extend from the chiasm to the thalamus
  • Afferent sensory nerves from the eye synapse with the second-order sensory neurones at the lateral geniculate nucleus in the thalamus
  • The sensory nerves radiate dorsally to the calcarine sulcus of the occipital lobe
  • Optic radiations loop either through the parietal lobe or through the temporal lobe (Meyer’s loop)
  • The optic radiations terminate in the calcarine sulcus of the occipital lobe where the cortical visual centre is situated
  • The calcarine sulcus is responsible for retinal image processing- images from both eyes are collated & a final image is formed
  • The image is inverted- so the brain has to re-invert the image
  • From the occipital visual centre, signals are sent to the frontal, parietal and temporal lobes to further make sense of the input information
How well did you know this?
1
Not at all
2
3
4
5
Perfectly
14
Q

Where is the visual cortex?

A
  • Calcarine cortex of the occipital lobe
How well did you know this?
1
Not at all
2
3
4
5
Perfectly
15
Q

What imaging is best for visual pathway?

A

MRI

How well did you know this?
1
Not at all
2
3
4
5
Perfectly
16
Q

How is the shape of the eyeball maintained?

A
  • Aqueous humour, fluid produced by the anterior and posterior chambers of the eyeball
    • The anterior chamber is the space between the cornea and the iris
    • Communicates with posterior chamber through the pupil
    • The posterior chamber is the space between the iris and lens
  • Aqueous humour is secreted by ciliary body and processes, fills the chambers of the eye, supports the shape of the eyeball by the pressure it exerts
How well did you know this?
1
Not at all
2
3
4
5
Perfectly
17
Q

Functions of aqueous humour?

A
  • Maintaining intraocular pressure and shape of globe
  • Provide nutrients and oxygen for ocular tissue including posterior cornea, trabecular meshwork, lens
  • Removal of metabolic by-products from intraocular cells
  • Facilitating passage of light from intraocular cells
How well did you know this?
1
Not at all
2
3
4
5
Perfectly
18
Q

How does aqueous humour drain?

A
  • Through iridocorneal angle (between iris and cornea)
  • Via trabecular meshwork (deteriorates with age- chronic open angle glaucoma)
  • Into canal of Schlemm
  • Blockage of this drainage à increase in intra-ocular pressure à glaucoma
How well did you know this?
1
Not at all
2
3
4
5
Perfectly
19
Q

Describe the accommodation reflex?

A
  1. Convergence- keeps image focused on fovea (highest visual acuity here)
  2. Pupillary constriction- to ensure image isn’t blurred
  3. Suspensory ligament relaxes- lens becomes fatter/ rounder (biconvex)- can focus image on fovea
How well did you know this?
1
Not at all
2
3
4
5
Perfectly
20
Q

What is presbyopia & briefly why does it happen?

A
  • Gradual loss of eye’s ability to focus on nearby objects
  • Insufficiency of accommodation
  • Age-related changes of the lens- decreased elasticity and increased hardness of the lens
How well did you know this?
1
Not at all
2
3
4
5
Perfectly
21
Q

Why does every normal eye have a small blind spot in the temporal visual field?

A
  • The optic nerve enters the retina at the optic disc
  • Here there are no photoreceptor cells
  • Hence blind spot
How well did you know this?
1
Not at all
2
3
4
5
Perfectly
22
Q

Describe the function of the lens?

A
  • Needed for normal vision
  • Refraction- change in direction of light, helps converge light onto the retina
  • Accommodation- maintaining focus on image as the distance varies, lens alters its shape through contraction or relaxation of the ciliary bodies
How well did you know this?
1
Not at all
2
3
4
5
Perfectly
23
Q

Why does cataracts occur, who is affected, & how do they present?

A
  • Opacification of the lens
    • Unilateral or bilateral
    • Lack of blood supply to lens means it is susceptible to damage from normal ageing and environmental insults eg UV light
    • Without transparency of the lens, light is unable to be refracted onto the retina to enable vision
  • Common as we age
  • Other causes- trauma, uveitis, scleritis, intra-ocular tumours, radiation, medications, systemic disease eg DM
  • Can be congenital
  • Hallmark feature: painless loss of vision
  • Symptoms: visual loss, blurred vision, poor night vision, sensitivity to light & glare, seeing ‘Halos’ around lights, polyopia (multiple images seen), reduction in colour intensity (lots of blues), changes in glasses prescription
  • Signs: reduced VA (snellens), loss of red reflex on ophthalmoscopy, white/ grey pupil due to opacification, nystagmus
How well did you know this?
1
Not at all
2
3
4
5
Perfectly
24
Q

Risk factors for developing cataracts?

A
  • Increasing age
  • Smoking
  • Alcohol
  • Diabetes
  • Steroids
  • Hypocalcaemia
How well did you know this?
1
Not at all
2
3
4
5
Perfectly
25
Q

How is cataracts diagnosed? Suggest some differential diagnoses

A
  • Clinically with ophthalmoscope
    • Loss or darkening of red reflex
    • Opacification seen within red reflex
    • Obscuration of ocular detail due to opacification and crystallisation
  • Slit-lamp confirms cataracts, type, and excludes other causes of visual loss such as glaucoma
  • Ddx- refractive error, corneal disease presbyopia, age related macular degeneration (ARMD), retinopathy, open angle glaucoma, uveitis
How well did you know this?
1
Not at all
2
3
4
5
Perfectly
26
Q

How is cataracts managed? Complications?

A
  • Definitive treatment: surgical replacement of lens
    • Phacoemulsification- breaking up diseased lens and aspirating the leftover content, lens remains in situ and new lens is placed into the capsule
    • Extracapsular cataract extraction- removal of the diseased nucleus and aspiration of the lens cortex, capsule left in situ and new rigid lens inserted, bigger incision required
  • Complications-
    • Immediate: endophthalmitis (bacterial or fungal infection of the intra-ocular fluid)- intravitreal abx required, can lead to loss of vision and the eye
    • Delayed- retinal detachment, macular degeneration, posterior capsule opacification
How well did you know this?
1
Not at all
2
3
4
5
Perfectly
27
Q

What is glaucoma?

A
  • NICE definition: glaucoma is a group of conditions with characteristic optic nerve head changes associated with corresponding visual field defects, +/- raised intraocular pressure
  • Recognised types of glaucoma:
    • Primary open-angle glaucoma
    • Angle closure glaucoma
    • Secondary glaucoma (due to uveitis, trauma etc)
    • Congenital glaucoma
How well did you know this?
1
Not at all
2
3
4
5
Perfectly
28
Q

How does glaucoma affect vision?

A
  • Loss of peripheral fields- tunnel vision
  • Night vision often worse
  • Damage to the optic nerve affects the peripheral vision first then gradually causes total sight loss if left untreated
How well did you know this?
1
Not at all
2
3
4
5
Perfectly
29
Q

What is the normal intraocular pressure? What is it created by?

A
  • 10-21 mmHg
  • This pressure is created by resistance to flow through the trabecular meshwork
How well did you know this?
1
Not at all
2
3
4
5
Perfectly
30
Q

What happens in open angle glaucoma?

A
  • Gradual increase in resistance through the trabecular meshwork
  • à difficult for aqueous humour to flow through the meshwork to exit the eye, hence pressure slowly builds up within the eye
  • Increased pressure in the eye causes cupping of the optic disc – normally the optic cup is less than half the size of the optic disc; in increased intraocular pressure the indent is wider- cupping of the optic disc occurs
How well did you know this?
1
Not at all
2
3
4
5
Perfectly
31
Q

Risk factors for primary open angle glaucoma?

A
  • Age
  • FHx
  • Black ethnic origin
  • Diabetes
  • Htn
  • Myopia (near sightedness)
How well did you know this?
1
Not at all
2
3
4
5
Perfectly
32
Q

Is there any screening for glaucoma?

A
  • No
  • However the following groups of people should have their eyes examined by an optometrist
    • Older age- >60 every 2 years, >70 every year (free through NHS)
    • FHx of glaucoma- people >40 with first-degree relative with open angle glaucoma have eye test annually (free through NHS)
    • People >40 of black African family origin- annual eye test (not through NHS)
How well did you know this?
1
Not at all
2
3
4
5
Perfectly
33
Q

How does primary open angle glaucoma present and how is it diagnosed?

A
  • Largely asymptomatic
  • Visual loss may occur but central vision is preserved until later in disease
  • Diagnosis via routine ophthalmic examination at optometrists
  • GPs may suspect if they visualise cupped discs during ophthalmoscopy
  • Diagnosis-
    • Ophthalmoscopy- cupped disc
    • Visual fields
    • Intraocular pressure- may or may not be elevated
How well did you know this?
1
Not at all
2
3
4
5
Perfectly
34
Q

How is intra-ocular pressure measured?

A
  • Goldmann Applanation Tonometry (GAT)- gold standard test
    • Special device mounted on slit lamp, makes contact with cornea and applies different pressures to the front of the cornea to get an accurate measurement
  • Non-contact tonometry- used by opticians
    • Shooting a puff of air at the cornea and measuring the corneal response to the air, less accurate, but gives helpful estimate for general screening purposes
How well did you know this?
1
Not at all
2
3
4
5
Perfectly
35
Q

How is open-angle glaucoma managed?

A
  • Management of ocular hypertension- treatment started at pressures >24 mmHg
    • First-line: topical prostaglandin analogue eg Latanoprost
      • These reduce IOP by increased uveosacral outflow
      • C/I: pregnancy & BF
      • S/E: brown pigmentation of iris, pigmentation of peri-ocular skin, local irritation
    • Topical beta blocker eg Timolol
      • These reduce IOP by lowering the production of aqueous humour
      • Caution asthma, COPD
      • S/E: local irritation
    • Lifetime monitoring is routine once treatment is commenced
    • Second-line: if first-line is unsuccessful or not tolerated
      • Switching to a drug in the other first-line drug class
      • Combining a topical prostaglandin analogue or prostamide with topical beta blocker
        Or switching to any of the following:
      • Topical sympathomimetic eg Brimonidine tartrate
        • Reduce production of aqueous humour and increased uveoscleral outflow
        • S/E: local irritation, dry mouth, unpleasant taste
      • Topical carbonic anhydrase eg topical Acetazolamide, topical Brinzolamide
        • Reduce aqueous humour secretion
      • Topical miotic eg Pilocarpine
        • Induce miosis- pulls iris away from trabecular meshwork allowing for improved outflow of aqueous humour
        • S/E: local irritation, myopia, vitreous haemorrhage, retinal detachment
  • Surgical options
    • Laser trabeculoplasty
      • Low-energy laser fired at trabecular meshwork
      • Increased drainage capacity of trabecular meshwork
    • Trabeculectomy
      • Creating a new channel from the anterior chamber through the sclera to a location under the conjunctiva
      • Causes a ‘bleb’ under the conjunctiva where the aqueous humour drains
      • The bleb is normally hidden by the upper eyelid
      • The fluid is reabsorbed from the bleb into the blood supply of the conjunctive, into the general circulation
    • Insertion of a drainage shunt
      • Small silicon flexible tube inserted through the sclera into the anterior chamber, allowing the aqueous humour to drain out into a tiny artificial reservoir attached to the end of the tube under the conjunctiva
How well did you know this?
1
Not at all
2
3
4
5
Perfectly
36
Q

Describe how acute angle closure glaucoma occurs

A
  • The iris bulges forward and seals off the trabecular meshwork from the anterior chamber
  • This prevents aqueous humour from being able to drain away
  • This results in continual build up of pressure in the eye, esp in the posterior chamber, which causes pressure behind the iris and worsens the closure of the angle
  • OPHTHALMIC EMERGENCY! Can lead to permanent vision loss
How well did you know this?
1
Not at all
2
3
4
5
Perfectly
37
Q

Describe some risk factors for developing closed angle glaucoma

A
  • Increasing age
  • Females > males
  • FHx
  • Chinese and East Asian ethnic origin- rare in black ethnicity (unlike open-angle glaucoma)
  • Shallow anterior chamber
  • Drugs
    • Adrenergic medications eg noradrenaline
    • Anticholinergic medications eg oxybutynin, solifenacin
    • Tricyclic antidepressants eg amitriptyline- anticholinergic effects
How well did you know this?
1
Not at all
2
3
4
5
Perfectly
38
Q

How do patients with closed angle glaucoma present?

A
  • Appear generally unwell in themselves
  • Short hx of severely painful red eye, blurred vision, halos around lights, associated headache, N&V
  • O/E- red eye, teary, hazy cornea, decreased VA, dilatation of affected pupil, fixed pupil size, firm eyeball on palpation
How well did you know this?
1
Not at all
2
3
4
5
Perfectly
39
Q

How to manage acute primary angle closure glaucoma?

A
  • Immediately admit- if there is a delay to admission do the following
    • Lie pt on their back w/o pillow
    • Give pilocarpine eyedrops
    • Give acetazolamide 500mg orally
    • Give analgesia and anti-emetic if required
  • In secondary care, various medical options to reduce IOP-
    • Pilocarpine
    • Oral or IV acetazolamide
    • Hyperosmotic agents eg glycerol or mannitol to increase osmotic gradient between the blood and fluid in the eye
    • Timolol- BB- reduces production of aqueous humour
    • Dorzolamide- CA inhibitor- reduced production of aqueous humour
    • Brimonidine- sympathomimetic- reduces production of aqueous humour & increased uveoscleral outflow
  • Definitive treatment-
    • Laser iridotomy- makes hole in iris to allow aqueous humour to flow from posterior chamber into anterior chamber, relieving pressure that was pushing the iris against the cornea, allows fluid to drain
How well did you know this?
1
Not at all
2
3
4
5
Perfectly
40
Q

How does pilocarpine work in acute angle closure glaucoma?

A
  • Acts on muscarinic receptors in sphincter muscles in iris and causes constriction of pupil
  • Therefore it is a miotic agent
  • Also causes ciliary muscle contraction
  • These 2 effects cause the pathway for the flow of aqueous humour from the ciliary body, around the iris and into the trabecular meshwork to open up
How well did you know this?
1
Not at all
2
3
4
5
Perfectly
41
Q

How do pts develop diabetic retinopathy (pathophysiology)?

A
  • Hyperglycaemia leads to damage to the retinal small vessels and endothelial cells
  • Increased vascular permeability leads to leakage from the blood vessels, blot haemorrhages and the formation of hard exudates (yellow/ white deposits of lipids in the retina)
  • Damage to blood vessel walls à microaneurysms and venous beading (walls of veins no longer straight/ parallel, look like strings of beads or sausages)
  • Damage to nerve fibres in the retina causes fluffy white patches to form on the retina- cotton wool spots
  • Intraretinal microvascular abnormalities (IMRA)- dilated and tortuous capillaries in the retina, can act as a shunt between the arterial and venous vessels in the retina
  • Neovascularisation- growth factors released in the retina causing new blood vessel development
    • May culminate in widespread vitreous haemorrhage causing sudden and complete visual loss
How well did you know this?
1
Not at all
2
3
4
5
Perfectly
42
Q

Risk factors for developing diabetic retinopathy?

A
  • Long period of exposure to hyperglycaemia
  • Htn
  • Ethnic minority background
  • Renal disease- proteinuria
  • Pregnancy
  • Rapid improvement of blood sugar levels
  • Hyperlipidaemia/ hypercholesterolaemia
How well did you know this?
1
Not at all
2
3
4
5
Perfectly
43
Q

How is diabetic retinopathy classified?

A
  • Non-proliferative (no new blood vessels)
    • Mild: microaneurysms
    • Moderate: microaneurysms, blot haemorrhages, hard exudates, cotton wool spots, venous beading
    • Severe: blot haemorrhages plus microaneurysms in 4 quadrants, venous beading in 2 quadrants, intraretinal microvascular abnormality in any quadrant
  • Proliferative (new blood vessels)
    • Neovascularisation- may lead to vitreous haemorrhage
    • Fibrous tissue forming anterior to retinal disc
    • More common in T1DM
  • Diabetic maculopathy
    • Macular oedema
    • Ischaemic maculopathy
    • Only sign may be change in VA
    • More common in T2DM
How well did you know this?
1
Not at all
2
3
4
5
Perfectly
44
Q

Screening for diabetic retinopathy?

A
  • All pts >age 12 with diabetes get a yearly eye test
How well did you know this?
1
Not at all
2
3
4
5
Perfectly
45
Q

How to treat diabetic retinopathy?

A
  • Laser photocoagulation
  • Anti-VEGF medications eg ranibizumab, bevacizumab
  • Vitreoretinal surgery (keyhole surgery on the eye) may be required in severe disease
How well did you know this?
1
Not at all
2
3
4
5
Perfectly
46
Q

Complications of diabetic retinopathy?

A
  • Neovascular glaucoma (secondary glaucoma)- acutely painful red eye, vision loss
  • Retinal detachment
  • Vitreous haemorrhage
How well did you know this?
1
Not at all
2
3
4
5
Perfectly
47
Q

What is Age-related macular degeneration?

A
  • AMD is the term applied to changes, without any other obvious precipitating cause, which occur in the central area of the retina (macula) in people age >50
How well did you know this?
1
Not at all
2
3
4
5
Perfectly
48
Q

Describe the changes that can occur in dry ARMD?

A
  • Drusen- collections of lipid beneath the retinal pigment epithelium (RPE) and within Bruch’s membrane
    • Some drusen can be normal. Normal drusen are small <63 micrometres and hard
    • Large and greater numbers of drusen can be early sign of macular degeneration
  • Retinal pigment epithelium (RPE) abnormalities- areas of hypo/ hyperpigmentation
  • Atrophy of retinal pigment epithelium
  • Degeneration of photoreceptors
  • Serous pigment epithelial detachment (PED) without neovascularisation
How well did you know this?
1
Not at all
2
3
4
5
Perfectly
49
Q

What are the 2 types of ARMD?

A
  • Dry- 90% cases- better prognosis
  • Wet- 10% cases- worse prognosis
    • Development of new vessels growing from the choroid layer into the retina
    • These vessels can leak fluid or blood, cause oedema, and more rapid vision loss
    • VEGF stimulates new vessel growth- target of medications to treat wet AMD
How well did you know this?
1
Not at all
2
3
4
5
Perfectly
50
Q

What are the layers of the macula?

A

4 key layers of the macula-

  1. Choroid layer at the bottom, contains blood vessels that provide the blood supply to the macula
  2. Bruch’s membrane
  3. Retinal pigment epithelium
  4. Photoreceptors (at the top)
How well did you know this?
1
Not at all
2
3
4
5
Perfectly
51
Q

What are some risk factors for age-related macular degeneration?

A
  • Age
  • Smoking
  • White or Chinese ethnicity
  • FHx
  • CVD, htn
  • Drug- aspirin
  • High fat diet
  • Other- sunlight exposure, blue eyes, females> males, previous cataract surgery
How well did you know this?
1
Not at all
2
3
4
5
Perfectly
52
Q

How do pts with AMD present?

A
  • Gradual worsening central visual field loss
  • Reduced visual acuity
  • Crooker or wavy appearance to straight lines
  • Wet AMD more acutely- loss of vision over days, full loss of vision over 2-3 yrs, often progresses to bilateral disease
How well did you know this?
1
Not at all
2
3
4
5
Perfectly
53
Q

What to expect on examination in AMD pts?

A
  • Snellen chart- reduced acuity
  • Scrotoma- central patch of vision loss
  • Amsler grid test- distortion of straight lines
  • Fundoscopy- Drusen
  • Slit lamp
  • Optical coherence tomography- cross-sectional views of layers of retina, to diagnose wet AMD
  • Fluorescein angiography- to see neovascularisation- wet AMD diagnosis if optical coherence tomography didn’t exclude it
How well did you know this?
1
Not at all
2
3
4
5
Perfectly
54
Q

How is AMD managed?

A
  • Dry AMD
    • No specific treatment
    • Avoid smoking, control BP, vitamin supplements
  • Wet AMD
    • Anti-VEGF medications- ranibizumab, bevacizumab, pegaptanib- injected directly into vitreous chamber once a month
      • See benefit after ~ 3 months
How well did you know this?
1
Not at all
2
3
4
5
Perfectly
55
Q

Who typically gets central retinal vein occlusion (CRVO)? (state the risk factors)

A
  • Patients > 45 secondary to retinal vein thrombosis
  • Diabetes
  • Hypertension
  • Hyperlipidaemia
  • Glaucoma
  • <45: clotting disorder- thrombophilia
  • Systemic inflammatory conditions eg SLE
  • Smokers
How well did you know this?
1
Not at all
2
3
4
5
Perfectly
56
Q

Describe the anatomy of the central retinal vein?

A
  • A short vein that runs through the optic nerve
  • 4 branched veins come together to form the central retinal vein
  • Leaves the optic nerve 10mm from the eyeball and drains blood from the capillaries of the retina into either superior ophthalmic vein or into the cavernous sinus directly
How well did you know this?
1
Not at all
2
3
4
5
Perfectly
57
Q

What happens when a retinal vein is blocked?

A
  • Pooling of blood in retina
  • Macular oedema & retinal haemorrhage
  • Retina tissue damage & loss of vision
  • Release of VEGF- neovascularisation
How well did you know this?
1
Not at all
2
3
4
5
Perfectly
58
Q

Describe the classification of central retinal vein occlusion?

A
  • Ischaemic- these pts are at risk of neovascular glaucoma. Some features suggesting ischaemia are:
    • Poor visual acuity
    • RAPD
    • Multiple dark deep intra-retinal haemorrhage
    • Multiple cotton wool spots
    • Thunder fundus
  • Non-ischaemic
How well did you know this?
1
Not at all
2
3
4
5
Perfectly
59
Q

How does CRVO present?

A
  • Sudden painless loss of vision
  • O/E- fundoscopy:
    • Flame and blot haemorrhages
    • Optic disc oedema
    • Macula oedema
How well did you know this?
1
Not at all
2
3
4
5
Perfectly
60
Q

How to manage retinal vein occlusion?

A
  • 2 main aims in the mx of retinal vein occlusion:
    • Identification of modifiable risk factors & their medical management eg treat glaucoma
    • Recognition & management of sight-threating conditions- treat macular oedema and prevent complications such as neovascularisation of the retina and iris and glaucoma
  • Laser photocoagulation
  • Intravitreal steroids
  • Anti-VEGF therapies
How well did you know this?
1
Not at all
2
3
4
5
Perfectly
61
Q

Describe the anatomy of the central retinal artery?

A
  • Branch of the ophthalmic artery, branch of the internal carotid artery arising immediately distal to the cavernous sinus
  • The ophthalmic artery gives rise to many branches which supply different components of the eye
  • The central retinal artery supplies the internal surface of the retina and the optic nerve
How well did you know this?
1
Not at all
2
3
4
5
Perfectly
62
Q

Causes of central retinal artery occlusion?

A
  • Most common cause: atherosclerosis
  • Giant cell arteritis, where vasculitis affecting the ophthalmic or central retinal artery causes reduced blood flow
How well did you know this?
1
Not at all
2
3
4
5
Perfectly
63
Q

Risk factors for developing central retinal artery occlusion?

A
How well did you know this?
1
Not at all
2
3
4
5
Perfectly
64
Q
A
  • Atherosclerosis- same risk factors as for cardiovascular disease- age, fhx, smoking, alcohol, htn, DM, poor diet, inactivity, obesity
  • Risk factors for GCA- age> 50, females, those already affected by GCA or PMR
How well did you know this?
1
Not at all
2
3
4
5
Perfectly
65
Q

How do pts with central retinal artery occlusion present?

A
  • Sudden painless loss of vision
  • Relative afferent pupillary defect
    • The pupil in the affected eye constricts more when light is shone in the other eye compared when it is shone in the affected eye
    • This occurs because the input is not being sensed by the ischaemic retina when testing the direct light reflex but is being sensed by the normal retina during the consensual light reflex
  • Fundoscopy
    • Pale retina with a cherry-red spot- this is the macula, which has a thinner surface that shows the red-coloured choroid below and contrasts with the pale retina
    • Retina is pale due to lack of blood perfusion
How well did you know this?
1
Not at all
2
3
4
5
Perfectly
66
Q

How to manage central retinal artery occlusion?

A
  • Immediate referral to Ophthalmology
  • Giant cell arteritis- ESR, temporal artery biopsy, high dose steroids
  • If symptoms suggest occlusion within 24 hrs, attempt to dislodge the embolus by:
    • Firm ocular massage through closed eyelids for 15 mins
    • Stat crushed acetazolamide 500mg PO and beta blocker
    • Offer anterior chamber paracentesis
    • Inhaling carbogen to dilate the artery
    • Sublingual isosorbide dinitrate to dilate the artery
  • Long term mx-
    • Treating reversible risk factors
    • Secondary prevention of CVD
How well did you know this?
1
Not at all
2
3
4
5
Perfectly
67
Q

What is anterior ischaemic optic neuropathy?

A
  • AION is caused by infarction of the optic nerve head
  • It may be non-arteritic or arteritic (GCA)
  • Sudden loss of vision
How well did you know this?
1
Not at all
2
3
4
5
Perfectly
68
Q

Risk factors for developing AION?

A
  • Male
  • Systemic arteriopathy
  • Age 40-60
  • Disc vessel crowding (disc w/o a cup, small hypermetropic discs, disc drusen)
How well did you know this?
1
Not at all
2
3
4
5
Perfectly
69
Q

How do you treat non-arteritic anterior ischaemic optic neuropathy?

A
  • There is no effective treatment
  • Review all vascular risk factors in casualty & exclude GCA
  • Re-check clinical findings, particularly fields- expect 3 lines of VA improvement in 30% pts
  • AION is unlikely to recur in the same eye after nerve fibre atrophy relieves disc vessel congestion
  • Review in 6 months, discharge if stable
How well did you know this?
1
Not at all
2
3
4
5
Perfectly
70
Q

What is retinal detachment?

A
  • Refers to the separation of the inner neurosensory retina from the underlying retinal pigment epithelium
  • This means vitreous fluid will accumulate in the subretinal space
  • The outer retina relies on the blood vessels of the choroid for blood supply
How well did you know this?
1
Not at all
2
3
4
5
Perfectly
71
Q

Risk factors for retinal detachment?

A
  • Posterior vitreous detachment
  • Diabetic retinopathy
  • Eye trauma
  • Retinal malignancy
  • Older age
  • Fhx
  • Myopia
  • Cataract surgery
How well did you know this?
1
Not at all
2
3
4
5
Perfectly
72
Q

When to suspect retinal detachment?

A
  • New onset floaters
  • New onset flashes
  • Sudden-onset painless & progressive visual field loss
  • Reduction in visual acuity, blurred or distorted vision, causing persistent and progressive visual loss
How well did you know this?
1
Not at all
2
3
4
5
Perfectly
73
Q

How to manage retinal detachment?

A
  • Reattach retina and reduce traction or pressure that may cause it to detach again-
    • Vitrectomy- removing relevant parts of vitreous body and replacing with oil or gas
    • Scleral buckling- using a silicone buckle to put pressure on the sclera so that the outer eye indents to bring the choroid inwards and in contact with the detached retina
    • Pneumatic retinopexy- injecting a gas bubble into the vitreous body and positioning the pt so that the gas bubble creates pressure that flattens the retina against the choroid and close the detachment
  • Repair retina tear-
    • Laser therapy
    • Cryotherapy
How well did you know this?
1
Not at all
2
3
4
5
Perfectly
74
Q

What is binocular vision?

A
  • The ability to use both eyes simultaneously so that each eye contributes to a common vision perception
How well did you know this?
1
Not at all
2
3
4
5
Perfectly
75
Q

What is strabismus? What terminology is used to describe strabismus?

A
  • AKA squint
  • An ophthalmic condition where the eyes do not properly align with each other when focusing to look at an object ie poor fusion
  • May be constant or intermittent and may vary with direction of gaze
  • Latent deviation- controlled by subconscious effort, only appears when binocular viewing is broken and the 2 eyes are not looking at the same object- unilateral cover test
    • -phoria
  • Manifest deviation- in certain situations such as fatigue, control is lost and the deviation becomes manifest. Present while the person views a target binocularly, with no occlusion in either eye
    • -tropia
  • When the eyes are convergent, the deviation is eso-, when the eyes are divergent, the deviation is -exo
  • Eso/exo - tropia/phoria
How well did you know this?
1
Not at all
2
3
4
5
Perfectly
76
Q

List 4 things to ask in a history of strabismus?

A
  • Diplopia? Is it monocular or binocular (only present with one eye open or both eyes), constant or intermittent, the position of gaze it occurs in, does it produce vertical/ horizontal/ tilted images?
  • Does it stop pt from doing things?
  • Is the deviation socially embarrassing?
  • Birth hx
  • Fhx of amblyopia, strabismus, refractive error
  • Previous visual acuity or ophthalmic testing results
  • PMH to include ophthalmic trauma, autoimmune or neoplastic conditions, toxin exposure, head trauma
How well did you know this?
1
Not at all
2
3
4
5
Perfectly
77
Q

How is the presence of strabismus confirmed clinically?

A
  • Cover test (single cover test) will detect manifest strabismus
    • Ask pt to focus on a target
    • Cover one eye and look at the other
    • If there is no shift in fixation of the uncovered eye, the patient has normal alignment = orthotropic
    • If there is a shift in fixation in the uncovered eye, then the patient has heterotropia
      • Exotropia: the eye is outwards at rest, and moves nasally when the opposite eye is covered
      • Esotropia: the eye is inwards at rest (nasal direction), and moves temporally when the opposite eye is covered
      • Hypertropia: the eye is upwards at rest and moves inferiorly when the opposite eye is covered
      • Hypotropia: the eye is downwards at rest and moves superiorly when the opposite eye is covered
  • Cover-uncover test (alternating cover test) will detect if a latent strabismus
    • If the above single cover test demonstrates no tropia
    • Cover one eye for 1-2 seconds, then quickly remove the occlude to restore binocular vision
    • The eye that was covered is observed (rather than the uncovered eye as in single cover test) for refixation movement
    • If a phoria is present, this eye will shift back to being orthotropia to re-establish sensory fusion with the other eye
      • Exophoria: the covered eye moves nasally when uncovered
      • Esophoria: the covered eye moves temporally when uncovered
      • Hyperphoria: moves inferiorly
      • Hypophoria: moves superiorly
How well did you know this?
1
Not at all
2
3
4
5
Perfectly
78
Q

Difference between concomitant and incomitant strabismus?

A
  • Concomitant
    • Angle of deviation is the same in all positions of gaze
    • Extraocular movements are full
    • Usually occurs in childhood
  • Incomitant
    • Angle of deviation is different in different positions of gaze
    • Extraocular movements are not full
    • Often compensated by involuntary adoption of a compensatory abnormal head posture – in this position, the angle of deviation is least & binocular vision may be achieved
    • It is mostly associated with defective eye movements eg third nerve palsy, thyroid eye disease
    • May be caused by restriction or paralysis of the extraocular muscles
      • Restriction: trapping or shortening of the muscle- due to orbital trauma, inflammation, tumour
      • Paralysis: loss of innervation to muscle- due to cranial nerve palsy(s)
    • Usually occurs in adults
How well did you know this?
1
Not at all
2
3
4
5
Perfectly
79
Q

Common causes of concomitant strabismus (squints) in children?

A
  • Refractive error
  • FHx
  • Prematurity
  • Developmental delay
  • Idiopathic
How well did you know this?
1
Not at all
2
3
4
5
Perfectly
80
Q

Risk factors for developing strabismus?

A
  • FHx of strabismus
  • Low birth weight
  • Premature birth
  • Maternal smoking
  • Vision problems such as farsightedness or cataract
  • Illness that affects muscles or nerves
  • Down syndrome
  • Head injury
How well did you know this?
1
Not at all
2
3
4
5
Perfectly
81
Q

Risk factors for developing strabismus in adulthood?

A
  • Eye or blood vessel damage
  • Loss of vision
  • Eye tumour or brain tumour
  • Graves’ disease
  • Stroke
  • Various muscle and nerve disorders
How well did you know this?
1
Not at all
2
3
4
5
Perfectly
82
Q

Consequence of untreated strabismus?

A
  • May be distressing for child and parent
  • Amblyopia is a significant long-term consequence
How well did you know this?
1
Not at all
2
3
4
5
Perfectly
83
Q

How can strabismus present itself in thyroid eye disease?

A
  • Presents as hypotropia or esotropia because the inferior rectus and the medial rectus muscles are the most commonly involved extraocular muscles
How well did you know this?
1
Not at all
2
3
4
5
Perfectly
84
Q

What is amblyopia?

A
  • Amblyopia is a condition of reduced visual acuity in the absence of any obvious structural abnormality or ocular disease
  • Usually unilateral but can be bilateral
    • Not necessarily an inherent problem with the eye itself
    • Diagnosis of exclusion; other explanation for visual loss should always be sought
    • Amblyopia is not improved by correcting any refractive error or pathological obstacle to vision eg cataract
  • During visual development, permanent neural connections between the eye and visual cortex are established. When there is a problem focussing in early childhood: the brain is not stimulated to develop correctly
How well did you know this?
1
Not at all
2
3
4
5
Perfectly
85
Q

What can amblyopia be caused by?

A
  • Most common cause- strabismus
  • Anisometropia (interocular difference in refractive error)
  • Refractive error, particularly if asymmetrical (anisometropia)
  • Stimulus deprivation- due to obscuration of the visual axis (ptosis, congenital cataract)
  • High Astigmatism
How well did you know this?
1
Not at all
2
3
4
5
Perfectly
86
Q

In a child, strabismus and amblyopia can be the sign of a serious problem. Which causes are of concern?

A
  • Retinoblastoma
    • If missed can cause blindness, loss of eye, death
  • Congenital cataract
    • If missed can cause blindness
  • To rule these out- all children are examined for red reflex at birth
    • If dark or white- something is blocking the light, refer to Ophthalmology immediately
How well did you know this?
1
Not at all
2
3
4
5
Perfectly
87
Q

Describe the different types of diplopia?

A
  • Monocular double vision/ blurry vision
    • Overlapping images with blurring
  • Horizontal binocular double vision
    • 2 identical images next to each other
  • Vertical binocular double vision
    • 2 identical images on top of each other
  • Oblique binocular double vision
    • 2 identical images at an angle to each other
How well did you know this?
1
Not at all
2
3
4
5
Perfectly
88
Q

Why do people with childhood squints not have diplopia?

A
  • Due to retinal suppression: the cortical inhibition of the visual sensation from 1 eye, when both eyes are open
  • The image is never consciously perceived the suppressed area of the retina
  • Depending on whether the patient is esoptropic or exotropic, either the nasal half or temporal half of the retina is suppressed
  • Essentially making pts operate monocularly
How well did you know this?
1
Not at all
2
3
4
5
Perfectly
89
Q

How can strabismus in childhood be treated?

A
  • Correct any refractive error- glasses
  • Reverse the amblyopia- part-time occlusion of the better seeing eye forcing the amblyopic eye to take up fixation and see
    • Non-compliance is a significant problem: if the amblyopia is severe, the occlusion may not be tolerated
  • Orthoptic management- may help control intermittent and latent deviations
  • Surgery- altering the pull of extraocular muscles for cosmetic reasons or for relief of abnormal head position (surgery won’t improve vision or reverse amblyopia and recurrence of squint later in life can happen)
How well did you know this?
1
Not at all
2
3
4
5
Perfectly
90
Q

What are the branches of the oculomotor nerve?

A
  • Superior branch- LPS, superior rectus
  • Inferior branch- MR, IR, IO
How well did you know this?
1
Not at all
2
3
4
5
Perfectly
91
Q

What happens to the eye in an oculomotor nerve palsy?

A
  • Most extraocular muscles are affected, besides superior oblique and lateral rectus
    • Inability to elevate, depress and adduct the eye
  • Affected eye is abducted (lateral rectus) and depressed (superior oblique)
  • ‘Down and out’
    • Superior oblique and lateral rectus act to give an unopposed exotropia and hypotropia plus a dilated pupil with paralysis of accommodation
  • Partial ptosis
  • Pupil may be involved and dilated- emergency- compressive pathology
How well did you know this?
1
Not at all
2
3
4
5
Perfectly
92
Q

Causes of third nerve palsy?

A
  • Can be congenital, but most often acquired
  • Microvascular- diabetes, htn- pupil-sparing third nerve palsy
  • PCA aneurysm- painful, unilateral 3rd nerve palsy- neuro emergency
  • SoL
  • Trauma
  • Migraine
  • Demyelinating disease
  • Infection
  • GCA (rare)
How well did you know this?
1
Not at all
2
3
4
5
Perfectly
93
Q

What can a painful, unilateral 3rd nerve palsy suggest? What to do in this situation?

A
  • Posterior communicating artery aneurysm- emergency neurological referral!
How well did you know this?
1
Not at all
2
3
4
5
Perfectly
94
Q

How to treat third nerve palsy?

A
  • Treat amblyopia in children <8 years old. Prisms may help in isolated muscle cases
  • Surgery only achieves a limited area of binocular single vision at best, the main aim is to improve the appearance.
  • Residual diplopia can be treated with an occlusive contact lens
How well did you know this?
1
Not at all
2
3
4
5
Perfectly
95
Q

What happens to the eye in a trochlear nerve palsy?

A
  • Superior oblique is affected only (SO is responsible for depressing and intorting the eye)
  • Diplopia
  • Head tilt away from site of lesion
How well did you know this?
1
Not at all
2
3
4
5
Perfectly
96
Q

Causes of 4th nerve palsy?

A
  • The trochlear nerve is the only nerve to leave brainstem backwards, before curling around and projecting anteriorly; this makes it increasing susceptible to damage from head trauma- back of head trauma eg RTA, horse riding accident, fall from height
  • Microvascular- DM, htn
  • Demyelinating disease
  • Tumour
  • Aneurysm
  • Congenital
How well did you know this?
1
Not at all
2
3
4
5
Perfectly
97
Q

What happens to the eye in an abducens nerve palsy?

A
  • Lateral rectus muscle paralysed (responsible for abduction of the eye)
  • Affected eye adducted by resting tone of medial rectus
    • Esotropia = inturned eye
  • If 6th nerve palsy + disc swelling or papilloedema à order emergency neuro imaging to rule out raised ICP
How well did you know this?
1
Not at all
2
3
4
5
Perfectly
98
Q

Causes of 6th nerve palsy?

A
  • The abducens nerve exits the pons and runs over the petrous temporal ridge, making it susceptible to damage from raised ICP
  • Microvascular- diabetes, htn
  • SoL
  • Trauma
  • Demyelinating disease
  • Infection (bacterial or viral)
How well did you know this?
1
Not at all
2
3
4
5
Perfectly
99
Q

How to treat 6th nerve palsy?

A
  • Most microvascular palsies and childhood post-viral palsies resolve spontaneously so offer prisms or occlusion for large incomitant deviations
  • If stable & symptomatic for >6 months, consider surgery
How well did you know this?
1
Not at all
2
3
4
5
Perfectly
100
Q

What is Duane’s syndrome?

A
  • A congenital strabismus due to abnormal development of the 6th cranial nerve
  • Anomalous innervation of the lateral rectus muscle, together with retraction of the globe, and narrowing of the palpebral fissure on adduction
  • Basically there are problems with abduction & adduction
How well did you know this?
1
Not at all
2
3
4
5
Perfectly
101
Q

What symptoms occur in Horner’s syndrome and why?

A
  • Partial ptosis- denervation of superior tarsal muscle
  • Miosis (pupillary constriction)- denervation of dilator pupillae muscle
  • Anhidrosis on ipsilateral side of face- denervation of sweat glands
How well did you know this?
1
Not at all
2
3
4
5
Perfectly
102
Q

Why might patients with thyroid eye disease experience diplopia? How can you manage this aspect of thyroid eye disease?

A
  • Due to inflammatory changes that is then followed by extraocular muscle fibrosis
  • Diplopia is commonly intermittent, vertical and is either up-gaze or primary position
  • Treatment- stop smoking as this worsens TED, prisms may help diplopia in the primary position, occlusion of one eye may be necessary in large strabismus
How well did you know this?
1
Not at all
2
3
4
5
Perfectly
103
Q

Describe the lymphatic drainage of the eyelids?

A
  • Lateral 2/3 of the lids drain to superficial parotid nodes
  • The medial lids drain to submandibular nodes
How well did you know this?
1
Not at all
2
3
4
5
Perfectly
104
Q

Causes of eyelid lumps?

A
  • Stye
  • Chalazion (Meibomian cyst)
  • Benign lid lesions, such as-
    • Epithelial lesions- squamous papilloma, epidermoid cyst
  • Vascular lesions eg capillary hemangioma- superficial strawberry ‘naevi’
  • Pigmented naevi
  • Xanthelasma- medial eyelids, middle-aged adults, hyperlipidaemia
  • Eyelid neurofibroma- isolated or associated w/ neurofibromatosis type I
  • Conjunctival granuloma
  • Neoplastic lid lesions
    • Pre-malignant disease eg acitinic keratosis (progresses to SCC), Bowen’s disease, lentigo maligna
    • Primary eyelid malignant disease: basal cell carcinoma
How well did you know this?
1
Not at all
2
3
4
5
Perfectly
105
Q

Clinical features of orbital disease?

A
  • Although a wide range of diseases can affect the orbit, they share many common features, by occupying volume within a confined space and damaging orbital structures
  • Hence the main clinical features of orbital disease are not specific to the underlying pathology
    • Proptosis
    • Visual loss
    • Double vision
    • Pain
How well did you know this?
1
Not at all
2
3
4
5
Perfectly
106
Q

Key things to ask in a history when exploring orbital pathology?

A
  • The P’s
    • Proptosis- bulging, prominent eye?
    • Pain- causes include inflammation, infection, acute pressure changed eg haemorrhage, bony/ neural invasion
    • Progression- symptoms may occur over minutes (eg haemorrhage), hours-days (inflammation, weeks-months (malignancy), months- years (benign tumour)
    • PMH- thyroid disease, malignancy, trauma, sinusitis
    • Perceptual visual changes- diplopia, blurring, altered colour perception, refractive change
    • Palpable or visible mass
    • Periorbital abnormalities- including sensory- paraesthesia, numbness, facial weakness, redness, tenderness, watering, lid anomalies
How well did you know this?
1
Not at all
2
3
4
5
Perfectly
107
Q

What are the main causes of disease affecting the orbit?

A
  • Thyroid eye disease
  • Orbital cellulitis
  • Trauma
  • Vascular abnormalities
  • Idiopathic inflammation
How well did you know this?
1
Not at all
2
3
4
5
Perfectly
108
Q

Differentials for acute red eye? (common exam Q)

A
  • Painless red eye
    • Conjunctivitis
    • Episcleritis
    • Subconjunctival haemorrhage
  • Painful red eye
    • Glaucoma
    • Anterior uveitis
    • Scleritis
    • Corneal abrasions or ulceration
    • Keratitis
    • Foreign body
    • Traumatic or chemical injury
How well did you know this?
1
Not at all
2
3
4
5
Perfectly
109
Q

What is blepharitis?

A
  • Common chronic inflammatory disorder of the margin of the lids which may cause persistent and annoying symptoms of irritation, watering and redness
  • Anterior blepharitis- inflammation of the base of the eyelashes- 2 main types that may co-exist:
    • Staphylococcal blepharitis- due to infection of the lid margins by staph aureus/ epidermidis
    • Seborrheic dermatitis
  • Posterior blepharitis- inflammation of the meibomian glands
    • = Meibomian gland dysfunction blepharitis
  • Mixed anterior & posterior blepharitis can occur
110
Q

Describe clinical features of blepharitis and how to manage it?

A
  • Characteristic clinical features- burning, itching, erythema, crusting of eyelids, worse in morning
  • Mx-
    • Eyelid hygiene measures and warm compress
      • Cotton tip buds to rub away scales, crusts, meibum and to reduce the bacterial load
      • Plugged meibomian glands should be express by pressure with a warm compress such as cotton wool balls or a soft cloth soaked in warm water or heatable masks
    • Topical abx- chloramphenicol if hygiene measures fail
    • Artificial tear drops for relief of symptoms, when necessary
    • Steroid eye drops for inflammatory side, used very sparingly
111
Q

What are some conditions commonly associated with blepharitis?

A
  • Dry eye syndrome
  • Seborrheic dermatitis
  • Rosacea
112
Q

Differentials for blepharitis?

A
  • Malignant tumours of eyelid- scc, bcc
  • Eczema
  • Infection
  • Infestation- pubic lice
  • Autoimmune- pemphigoid
113
Q

Suggest some causes of contact dermatitis of the eyelids?

A
  • Makeup
  • Skin cleansers
  • Hair shampoo/ sprays
114
Q

Features of contact dermatitis on the eyelids?

A
  • Erythema, swelling, scales of eyelid skin
115
Q

What are the types of stye? How do they resolve/ how to manage in GP?

A
  • Acute onset painful localised swelling near the eyelid margin that develops over several days
  • Hordeolum externum- infection of glands of Zeis (sebaceous glands at base of eyelashes) or glands of Moll (sweat glands at base of eyelashes)
  • Hordeolum internum- infection of Meibomian glands- deeper, more painful, may point inward towards the eyeball under the eyelid
  • Symptoms resolve within 5-7 days once the stye spontaneously ruptures or been drained
  • Mx in GP setting- pluck eyelash from infected follicle to facilitate drainage, I&D by someone with suitable expertise
  • Admit to hospital if suspecting periorbital or orbital cellulitis
116
Q

What is a chalazion? How to treat?

A
  • Meibomian gland becomes blocked and swells up
  • Painless swelling of eyelid
    • Can be tender or red
    • More common on upper eyelid than lower eyelid
    • Usually 2-8mm in diameter
    • One or both eyes can be affected
    • More than one cyst can be present
  • Treatment- hot compress twice daily, analgesia
  • Rarely if this fails- surgical I&D
117
Q

What is ptosis and what causes it?

A
  • Droopy upper eyelid- ptosis describes a low position of the upper lid on the globe
  • It is in a diagnosis
  • Entropion (turning inwards) and ectropion (turning outwards) both cause watering, as a result of eyelashes irritating the ocular surface (entropion) or impairment of tear collection by the eyelid puncta (ectropion)
  • May be unilateral or bilateral
  • Causes-
    • Congenital Horner’s syndrome
    • Myasthenia gravis
    • Trauma
    • Oculomotor nerve disease
    • Pseudoptosis- aberrant facial nerve regeneration, enophthalmos, hypotropia, hyperglobus
118
Q

What is entropion? What are the causes and management?

A
  • Entropion is a turning inward of the lid margin such that normal eyelashes abrade the eyeball
  • May affect upper or lower lid
  • Causes
    • Involutional (aging) changes
    • Conjunctival scarring (cicatricial)
    • Spasm of orbicularis oculi
  • Mx
    • Initial- taping down eyelid to prevent eye drying out
    • Regular eyedrops
    • Surgical intervention- lid tightening
119
Q

What is ectropion? What are the causes & management?

A
  • Ectropion is turning outwards of the eyelid margin
  • Causes
    • Aging (due to collagen laxity)
      • Age-related ectropion is common, but exclude other causes
    • Paralytic: facial nerve palsy
    • Eyelid skin scarring eg from eczema or from previous lower lid surgery
    • Mechanical: lower lid mass eg meibomian cyst or bulky eyelid tumours
  • Mx- surgery
120
Q

What is trichiasis? How to treat?

A
  • Inward growth of eyelashes
  • Results in pain and can result in corneal damage and ulceration
  • Mx- specialist removes the eyelash (epilation)
  • Rare cases may require electrolysis, cryotherapy, laser treatment to prevent the lash regrowing
121
Q

What is necrotising fasciitis of the eyelid, risk factors for developing NF, and how to manage?

A
  • A destructive microbial infection of the soft tissue which can cause precipitous tissue destruction, septicaemia, DIC and death
  • Causative organisms eg neg bacilli, strep, staph
  • Risk factors- advancing age, injury eg insect bite, immunosuppression including diabetes
  • Mx- immediate IV benzylpenicillin 2.4g qds and IV clindamycin 600mg qds
  • Debride necrotic tissue down to a healthy (bleeding) bed
  • Repeat as necessary
  • Send tissue for culture & sensitivity
  • Subsequent reconstructive approaches
122
Q

What is conjunctivitis?

A
  • Inflammation of the conjunctiva
  • 3 main types- bacterial, viral, allergic
123
Q

How does conjunctivitis present?

A
  • Unilateral or bilateral
  • Red eyes
  • Bloodshot
  • Itchy or gritty sensation
  • Discharge from eyes
  • None of the following: pain, photophobia, reduced VA (blurry vision due to discharge but when the discharge is cleared the vision is normal)
  • Bacterial- purulent discharge, inflamed conjunctiva, typically worse in morning when the eyes may be stuck together, starts in 1 eye and can spread to the other, highly contagious
  • Viral- common, clear discharge, other viral features- dry cough, sore throat, blocked nose. Tender pre-auricular lymph nodes. Contagious
  • Allergic- swelling of conjunctival sac and eye lid, significant watery discharge, itchy
124
Q

How to manage the different types of conjunctivitis?

A
  • Usually resolves within 2 weeks without treatment
  • Advice- good hygiene to avoid spreading (don’t share towels or rub eyes, and regular hand washing), avoid contact lenses use, clean the eyes with cooled boiled water and cotton wool to help clear the discharge
  • Bacterial- consider abx eye drops (although not always required)- chloramphenicol, fusidic acid eye drops
  • Allergic- oral/ topical antihistamines, topical mast-cell stabilisers (require several weeks before showing benefit)
125
Q

You have a 1-month-old patient with conjunctivitis, what will you do & why?

A
  • Pts < 1 month- urgent ophthalmology review because neonatal conjunctivitis can be associated with gonococcal infection and cause loss of sight and serious complications such as pneumonia
126
Q

What is episcleritis? Who is affected?

A
  • Benign, self-limiting inflammation of the episcleral, the outermost layer of the sclera
  • Episcleral is just underneath the conjunctiva
  • Often associated with inflammatory disorders such as rheumatoid arthritis, IBD
127
Q

How does episcleritis present?

A
  • Acute onset unilateral symptoms
  • Typically painless/ mild pain
  • Segmental redness (rather than diffuse)
    • Patch of redness on lateral sclera
  • Foreign body sensation
  • Dilated body sensation
  • Watering of eye
  • Discharge
128
Q

How to manage pts with episcleritis?

A
  • Usually self-limiting & resolves within 1-4 weeks
  • Lubricating eye drops may aid symptoms
  • Simple analgesia
  • Cold compress
  • Safety net advice
  • Systemic NSAIDs- naproxen or topical steroid eye drops in severe cases
129
Q

What is scleritis?

A
  • Inflammation of the full thickness of the sclera
  • More serious than episcleritis
130
Q

What systemic conditions is scleritis associated with?

A
  • Rheumatoid arthritis
  • SLE
  • IBD
  • Sarcoidosis
  • Granulomatosis with polyangiitis
131
Q

How do pts with scleritis present?

A
  • Acute onset of symptoms, ~50% bilateral
  • Severe ocular & periocular pain (in comparison to episcleritis- painless or mild pain)
    • Particularly bad at night
  • Pain w/ eye movement
  • Redness
  • Photophobia
  • Eye watering
  • Reduced VA
  • Abnormal pupil reaction to light
  • Globe tenderness to palpation of the eye
132
Q

How to manage scleritis?

A
  • Emergency referral if in GP
  • NSAIDs
  • Steroids
  • Immunosuppression in relation to systemic disease
133
Q

Causes of subconjunctival haemorrhage?

A
  • A bleed into the subconjunctival space
  • Most idiopathic
  • Ask about htn, Valsalva (coughing. Constipation, heavy lifting, bleeding, anticoagulants, antiplatelets, trauma
134
Q

Name some factors that can contribute to developing a subconjunctival haemorrhage?

A
  • Hypertension
  • Bleeding disorders eg thrombocytopenia
  • Whooping cough
  • Medications- warfarin, NOACs, antiplatelets
  • Non-accidental injury
135
Q

What does a subconjunctival haemorrhage look like O/E?

A
  • Area of localised, well-demarcated haemorrhage in one eye
  • May be a hx of a precipitating event eg coughing fit or heavy lifting
  • No pain
  • No reduction of VA
  • Normal pupil reactions
  • No corneal staining
136
Q

Do we treat pts with sub conjunctival haemorrhage?

A
  • No
  • Reassure the haemorrhage will fade over 2 weeks
  • If there is a foreign body sensation lubricating eye drops may help with the symptoms
137
Q

What are corneal abrasions?

A
  • Scratches or damage to the cornea
  • A shearing force breaches the epithelium to expose the subepithelial nerve plexus
138
Q

Symptoms of corneal abrasion?

A
  • Painful red eye
  • Photophobia
  • Foreign body sensation
  • Tearing
  • Watering eye
  • Hx of contact lenses or FB
  • Blurring vision
  • Photophobia
139
Q

Causes of corneal abrasion?

A
  • Contact lenses
  • Foreign body
  • Fingernails
  • Eyelashes
  • Entropion (inward turning eyelid)
140
Q

How to diagnose a corneal abrasion?

A
  • Fluorescein stain- yellow orange colour- collects in abrasions or ulcers, highlighting them
  • Slit light examination for more significant abrasions
141
Q

How to manage corneal abrasion?

A
  • Simple analgesia
  • Lubricating eye drops
  • Abx eye drops eg chloramphenicol
  • Bring the pt back after 1 week to check it has healed
  • Cyclopentolate eye drops to dilate the pupil & improve significant symptoms such as photophobia- not usually necessary
142
Q

How long do corneal abrasions take to heal?

A
  • Approx. 2-3 days if they are uncomplicated
143
Q

What is keratitis?

A
  • Inflammation of the cornea
144
Q

What causes keratitis?

A
  • Viral infection- herpes simplex
  • Bacterial infection- pseudomonas or staphylococcus
  • Fungal infection- candida or aspergillus
  • Contact lens acute red eye (CLARE)
  • Exposure keratitis caused by inadequate eyelid coverage eg eyelid ectropion
145
Q

What is the most common cause of keratitis?

A
  • Herpes simplex infection
146
Q

What happens in Herpes keratitis and how do patients present?

A
  • Can occur as a primary infection (first eye manifestation) or recurrent
  • May affect one or more of the 3 corneal layers
    • Epithelial layer- most common
    • Stromal layer
      • Complications- stromal necrosis, vascularisation, scarring, corneal blindness
  • Presentation-
    • Painful red eye
    • Photophobia
    • Vesicles around the eye
    • FB sensation
    • Watering eye
    • Reduced VA (can be subtle or significant)
    • A hazy cornea or cream opacity (suggests stromal keratitis)
    • Reduced corneal sensation
147
Q

How to diagnose herpes keratitis?

A
  • Staining with fluorescein- shows a dendritic corneal ulcer (branching and spreading of the ulcer)
  • Slit lamp exam to find and diagnose keratitis
  • Corneal swabs or scrapings used to isolate the virus using viral culture or PCR
148
Q

How to manage herpes keratitis?

A
  • Epithelial HSV- debride loose epithelium using topical anaesthetic and a cotton wool swab
  • Acyclovir- topical or oral
  • Ganciclovir eye gel
  • If there is ulceration of the cornea, then treat prophylactically with topical chloramphenicol
  • Topical steroids may be indicated for stromal keratitis (not epithelial)
  • Corneal transplant may be required for scarring of the cornea from stromal keratitis
149
Q

Describe some complications of ocular HSV?

A
  • Corneal scarring and visual impairment
  • Corneal perforation
  • Secondary infection with bacteria or fungi
  • Systemic infection such as aseptic meningitis, encephalitis, or hepatitis
150
Q

Suggest some complications of wearing contact lenses

A
  • Bacterial keratitis
  • Other types of keratitis
  • Corneal abrasion
  • Contact kens associated red eye (CLARE)
  • 3 and 9 O clock staining
  • Inferior closure stain
151
Q

What is the uvea? What is uveitis?

A
  • The pigmented middle layer of the eyeball
  • 3 segments: the iris, ciliary body and choroid
  • Uveitis is inflammation of the uveal tract
    • Inflammation of nearby tissues eg retina, optic nerve, vitreous humour can also occur
152
Q

How is uveitis classified?

A
  • Anterior uveitis- anterior segment
    • Pain, redness, photophobia
  • Intermediate uveitis- inflammation of vitreous
    • Floaters, blurry vision, vitreous opacities, snowballs (aggregates of inflammatory cells in the vitreous humour posterior to the lens)
  • Posterior uveitis- inflammation of retina or choroid
  • Panuveitis- anterior chamber, vitreous and retina or choroid
153
Q

What is anterior uveitis?

A
  • Primary site of inflammation: anterior chamber of the uvea
  • The anterior chamber becomes infiltrated by neutrophils, lymphocytes, macrophages
    • O/E: deposits of white blood cells are often seen in the anterior chamber
  • Causes- usually autoimmune, infection, trauma, ischaemia, malignancy
  • Acute or chronic
154
Q

What is acute anterior uveitis associated with?

A
  • Ankylosing spondylitis
  • Inflammatory bowel disease
  • Reactive arthritis
155
Q

What is chronic anterior uveitis associated with?

A
  • Sarcoidosis
  • Syphilis
  • Lyme disease
  • Tuberculosis
  • Herpes virus
156
Q

How does uveitis present?

A
  • Pain or dull ache
  • Red eye
  • Diminished or blurred vision
  • Watering of the eye
  • Photophobia- due to ciliary muscle spasm
  • Flashes or floaters
  • Unreactive or distorted pupil
  • Ciliary flush- a ring of red spreading from the cornea outwards
  • Excessive tear production
  • Abnormally shaped pupil due to posterior synechiae (adhesions)
157
Q

How does anterior uveitis present?

A
  • Onset over hours or days of redness, pain, photophobia
  • Usually unilateral but may be bilateral
158
Q

Differentials for uveitis?

A
  • Acute glaucoma
  • Keratitis
  • Scleritis
  • Ocular trauma
159
Q

How to treat uveitis?

A
  • Steroids-topical drops
    • Systemic steroids may be a possibility if pts recur
  • Cycloplegic-mydriatic medications eg cyclopentolate or atropine eye drops- antimuscarinics, block the action of the iris sphincter muscles & ciliary body. Dilate pupils and reduce pain associated with ciliary spasm by stopping the action of the ciliary body
    • Cycloplegic = paralysing ciliary muscles
    • Mydriatic = dilating pupil
  • Suspecting infection- topical abx (chloramphenicol)
  • Immunosuppressants eg DMARDs and TNF inhibitors
  • Laser therapy, cryotherapy, surgery (vitrectomy) are also options in severe cases
160
Q

What are some possible problems affecting the front of the eye in the ICU?

Suggest how we can recognise and manage them.

A
  • Direct injury to cornea- most often a superficial corneal abrasion
    • Red eye, seen using fluorescein eye drops and a blue light
    • Give chloramphenicol ointment for 5-7 days qds
  • Exposure keratopathy
    • Dryness of the cornea due to incomplete lid closure allowing tear evaporation & consequent failure of the tears to spread adequately across the eye surface
    • Red eye
    • Lid taping if there is unwanted corneal exposure
  • Chemosis (conjunctival swelling)
    • Risk factors- compromise venous return from ocular structures (eg from tight endotracheal tube taping), states of generalised oedema (eg fluid overload), incomplete eyelid closure can also predispose chemosis
    • Chemosis can cause impaired eyelid closure
  • Microbial conjunctivitis and keratitis
    • Respiratory secretions are thought to be the major source of ocular surface infection, with aerosols from tracheal suctioning and direct contact from suction catheter both being implicated
    • If endotracheal suctioning is done from the side of the pt (rather than at the head) and with the eyes covered, pseudomonas infection rates can be reduced
    • Conjunctivitis: look for a red sticky eye, take swabs of the eye discharge, remove discharge by bathing eyelids with warm water, using a separate gauze for each eye. Use chloramphenicol ointment (rather than drops to aid lubrication) qds for 5-7 days
    • Microbial keratitis: the damaged cornea is especially vulnerable to bacterial invasion which can occur rapidly. Most cases are due to bacteria & appear as a red eye, corneal ulcer. Seek urgent ophthalmology referral
161
Q

How to identify those at greatest risk of corneal injury in the ITU? How to protect the eyes of ITU pts?

A
  • Assess eyelid closure- is the lid completely closed, or is there some conjunctival exposure (any white bit visible) or any corneal exposure (even a tiny amount)?
  • Protective measures-
    • Manual closure of the eyes or taping the eyes shut- although this can be distressing to relatives and repeated removal of tape can cause facial skin or eyelid injury or irritation so try not to use tape
    • Liberal use of lubricants into the eye- ointment > drops, because drops don’t last long enough
162
Q

Describe the causes of sudden loss of vision

A
  • Vascular- AION (Anterior Ischaemic Optic Neuropathy), retinal venous and arterial occlusion
    • Amaurosis fugax, may represent a form of TIA (give 300mg aspirin)
    • Altitudinal field defects are often seen- ‘curtain coming down’
    • Ischaemic optic neuropathy due to occlusion of the short posterior ciliary arteries, causing damage to optic nerve
    • CRVO: severe retinal haemorrhages often seen on fundoscopy
    • CRAO: due to thromboembolism (atherosclerosis) or due to arteritis (GCA), cherry red spot on pale retina, afferent pupillary defect
  • Retinal detachment
    • Features of vitreous detachment, floaters or flashes or light
  • Vitreous haemorrhage
    • Sudden visual loss, dark spots
    • Diabetes, bleeding disorders, anticoagulants
  • Wet Age-Related Macular Degeneration (ARMD)
163
Q

State some causes of chronic visual loss

A
  • Cataract
  • Dry ARMD
  • Glaucoma
164
Q

Suggest some ocular manifestations of child abuse?

A
  • Periorbital trauma- ecchymosis, lid oedema, orbital fractures
  • Anterior segment trauma- iris prolapse, corneal laceration, cataract
  • Posterior segment trauma- vitreous haemorrhage, retinal detachment, optic nerve avulsion
  • Associated brain injury resulting in nystagmus, cortical blindness, encephalopathy, cranial nerve palsies
165
Q

Features of shaken baby syndrome?

A
  • Triad: retinal haemorrhages, subdural haematoma, encephalopathy
  • Retinal haemorrhages are the cardinal manifestation
  • Splitting of the retinal layers
  • Retinal haemorrhages may subside over time so prompt retinal photography is recommended at the time of discovery of retinal haemorrhage on dilated fundoscopy to record findings as a baseline for future comparison
166
Q

What is a retinoblastoma?

What is the average age of diagnosis, and briefly describe the pathophysiology?

A
  • The most common ocular malignancy found in children
  • Average age of diagnosis is 18 months
  • Pathophysiology-
    • Autosomal dominant
    • Caused by a loss of function of the retinoblastoma tumour suppressor gene on chromosome 13
    • ~10% cases hereditary
167
Q

When to suspect a retinoblastoma in GP?

A
  • Absent red reflex
    • Replaced by a white pupil (leukocoria) from a large posterior retinal tumour mass
  • Strabismus
  • Visual problems
168
Q

How to manage retinoblastoma?

A
  • Local therapy
    • Enucleation for advanced local tumours or after treatment failure
  • Systemic chemotherapy- multi agent chemotherapy used alone or with local treatments as primary or salvage therapy
  • External beam radiotherapy now abandoned due to long-term morbidity and mortality risks
169
Q

What is hypertensive retinopathy?

A
  • Describes the damage to the small blood vessels in the retina relating to systemic hypertension
  • This can be a result of years of chronic htn or can develop quickly in response to malignant htn
170
Q

Describe the signs of hypertensive retinopathy?

A
  • Silver wiring or copper wiring where the walls of the arterioles become thickened & sclerosed causing increased reflection of the light
  • Arteriovenous nipping where the arterioles cause compression of the veins where they cross, again due to sclerosis & hardening of the arterioles
  • Cotton wool spots- due to ischaemia & infarction in the retina causing damage to nerve fibres
  • Hard exudates- caused by damaged vessels leaking lipids into the retina
  • Retinal haemorrhages- damaged vessels rupture & release blood into retina
  • Papilloedema- ischaemia to the optic nerve resulting in optic nerve oedema & blurring of the disc margins
171
Q

How to manage hypertensive retinopathy?

A
  • Control BP
  • Control other risk factors such as smoking and blood lipid levels
172
Q

Most common form of ocular involvement in HIV?

A
  • HIV retinopathy
173
Q

Causes of optic neuritis?

A
  • Multiple sclerosis- the commonest associated disease
  • Diabetes
  • Syphilis
174
Q

Who is typically affected by MS-associated optic neuritis?

A
  • Young women- average age 31
  • Who spent their childhood in temperate latitudes
  • Those with MS
175
Q

How does MS associated optic neuritis present?

A
  • Unilateral decrease in VA over hrs or days
  • Poor discrimination of colours, ‘red desaturation’
  • Pain worse on eye movement
  • Relative afferent pupillary defect
  • Central scrotoma
176
Q

How to manage optic neuritis?

A
  • High dose steroids
  • Recovery usually takes 4-6 weeks
  • Prognosis-
    • MRI: if >3 white matter lesions, 5 year risk of developing MS is 50%
177
Q

What is sarcoidosis?

A
  • A systemic granulomatous disease commonest in Asian and Afro-Caribbean pts
  • Multiple sites affected- in particular, lungs, lymph nodes, skin, liver, eyes
  • The eyes are affected in 30-60% of cases
178
Q

Describe some ocular pathologies that sarcoidosis can cause?

A
  • Sarcoidosis can lead to a number of ocular pathologies- mainly uveitis (bilateral)
    • Anterior- pain, redness, photophobia
    • Intermediate- inflammation of ciliary body, floaters, blurry vision
    • Posterior- choroiditis- may involve retinal vasculitis, floaters, visual loss
  • Keratoconjunctivitis sicca
  • Adnexal granuloma
  • Secondary glaucoma
179
Q

How to manage patients with sarcoidosis with ocular involvement?

A
  • Steroids as appropriate
  • Neovascularisation secondary to ischaemia or inflammation may occur; so laser or steroids may be used
180
Q

What is ocular TB? Give some examples of ocular manifestations of TB

A
  • Defined as an infection by MTB in the eye, around the eye or on its surface
  • Not usually associated with clinical evidence of pulmonary TB
  • The conjunctiva, cornea, sclera are sites of primary ocular involvement
  • Ocular manifestations of TB are varied and include scleritis, interstitial keratitis, corneal infiltrates, anterior chamber and iris nodules, anterior uveitis
181
Q

What is thyroid eye disease?

Which gender does it predominantly affect?

A
  • Idiopathic autoimmune disorder resulting in an active orbital inflammatory phase (months-years) and subsequent inactive fibrotic phase, largely involving extraocular muscles and orbital fat/ connective tissues
  • Affects more women than men
182
Q

Clinical features of thyroid eye disease?

A
  • Symptoms-
    • Gritty or watery eyes
    • Puffy eyelids
    • Ocular pressure or pain
    • Angry looking eyes
    • Bulging eyes
    • Diplopia
    • Visual loss
    • Field loss
    • Dyschromatopsia- colour vision problems
    • Depression
  • Signs-
    • Proptosis- unilateral reflecting the asymmetric muscle involvement
    • Lid lag on downgaze (von Graefe sign)
    • Lid oedema
    • Increased IOP- glaucoma may result from decreased episcleral venous outflow, because of restrictive myopathy, intraocular pressure may rise >8mmhg on up-gaze
    • Strabismus- hypotropia or esotropia because the inferior rectus and medial rectus muscles are the most commonly involved extraocular muscles
    • Optic nerve compression may occur with seemingly mild proptosis – often without visible optic nerve oedema
      • Document visual acuity, colour vision, presence/ absence of a relative afferent pupillary defect during each visit
183
Q

What diseases is Graves’ disease associated with?

A
  • Pernicious anaemia
  • Diabetes- very important due to steroid treatments
  • Autoimmune adrenal insufficiency
  • Vitiligo
  • Systemic sclerosis
  • Sjogren syndrome
  • Rheumatoid arthritis
  • SLE
184
Q

How to assess patients with thyroid eye disease?

A
  • General health
  • Ocular health
  • Hx- Fhx, SMOKING, occupation, hyper/hypothyroid, compliance to meds, which meds and how often they take them, symptoms, old images
  • Examination- VA, istihara colour vision assessment, exophthalmometry, orbital tension, assess lid lag, lid retraction, corneal exposure- ulcer?
  • Orthoptic assessment- field of binocular single vision, field of uniocular fixation, Hess chart, visual field study, take pictures
185
Q

Describe the approach to management of thyroid eye disease?

A
  • Clinical activity score- done when you first see them and when you see them again to see improvements/ decline
    • Pain- painful oppressive feeling on or behind globe, pain on attempted up/ down/ side gaze
    • Redness- of eyelids or conjunctiva
    • Swelling- of lids, chemosis, swollen caruncle, increase in proptosis of >2mm in 1-3 months
    • Impaired function- decrease in eye movements of 5 degrees in 1-3 months, or decrease in VA on Snellens or 1 or more lines in 1-3 months
  • > or including 4= benefit from immunosuppression
  • <4= risks outweigh the benefits of immunosuppression
  • Immunosuppression options:
    • IV or oral corticosteroids if active inflammation coexists with moderate or severe TED
  • Optimise endocrine control
  • STOP SMOKING
  • Avoid radioiodine
  • Lubricants
  • Ocular hypotensives
  • Radiotherapy
  • Surgery- orbital decompression firstly, strabismus surgery, lid surgery (upper/ lower retractions)
186
Q

Imaging for stroke?

A
  • Non-contrast CT
187
Q

What is giant cell arteritis?

A
  • Sight-threatening vasculitis
  • Characterised by granulomatous inflammation of medium and large-sized arteries (vessel walls)
    • Granulomas present on microscopy- collection of macrophages
188
Q

How do pts with GCA present?

A
  • Symptoms-
    • New headache
    • Scalp tenderness
    • Loss of appetite
    • Weight loss
    • Limb girdle pain (worse in morning, relieved by movement)
    • Ischaemic jaw pain on chewing
    • Prodromal episodic transient visual loss, often on standing due to poor perfusion, occurs in 10%
    • Ophthalmoplegia may occur (paralysis/ weakness of extraocular muscles)
  • Signs-
    • Thickened temporal arteries, may be non pulsatile & tender
    • RAPD
    • Pale disc swelling +/- flame haemorrhages
    • VA < count fingers
    • Central retinal artery occlusion (pale retina w/ cherry red spot) and retinal cotton wool spots may occur
    • Optic nerve ischaemia may be retrobulbar
189
Q

How is a diagnosis of giant cell arteritis made?

A
  • When 3 of the following criteria are met-
    • Age at disease on set >50
    • New headache
    • Temporal artery abnormality
    • Elevated ESR (>50)
    • Abnormal temporal artery biopsy
190
Q

How to manage giant cell arteritis?

A
  • High dose corticosteroids
    • With visual symptoms: 60-100mg one off dose prednisolone until further ophthalmic assessment. Alternatively, IV methylprednisolone 500-1000mg for 3 days then oral prednisolone ~60mg daily
    • Without visual symptoms: 40-60mg prednisolone daily
  • If there isn’t a rapid response to steroids, re-assess the diagnosis of GCA
  • Steroid-related complications- steroid-induced hyperglycaemia, mood changes, insomnia, GI bleeding, immunosuppression, weight gain, Cushingoid appearance, osteoporosis, adrenal cortex suppression
  • Review at 1 week and then monthly w/ repeat ESR
  • When pt is asymptomatic & the ESR has significantly fallen- start a steroid-reducing regimen
191
Q

Describe the main complications of GCA?

A
  • The major one is visual loss: 20% pts, little chance of recovery once lost
  • Larger arterial complications: aortic aneurysm, aortic dissection, large artery stenosis
  • CVD
  • Steroid-related complications
192
Q

Describe some broad groups of eyelid injury?

A
  • Lacerating trauma
  • Blunt trauma
  • Chemical injury
193
Q

Suggest 4 questions to ask when taking a history from a patient with eyelid trauma?

A
  1. The time & nature of alleged assault or foreign body as well as the distance and trajectory
  2. The foreign body material and structure and whether all parts have been recovered
  3. Other ocular symptoms
  4. ENT symptoms including epistaxis, CSF rhinorrhoea
194
Q

What is the most common orbital injury?

How do you confirm the diagnosis?

A
  • Blowout fracture
    • Typically involving the floor or medial wall
  • X-ray of the face is a first line ix
  • A definitive diagnosis is confirmed by CT scan of the orbit
195
Q

Key diagnostic features of an orbital blowout fracture?

A
  • Diplopia on upward gaze
  • Derangement of globe position
  • Intercanthal distance increased
  • Oculovagal symptoms- bradycardia, hypotension, N&V)
  • Visual disturbance
  • Peri-orbital ecchymosis (bruising)
  • Peri-orbital oedema
  • Nerve sensory loss- numbness over cheek, lower eyelid, upper lip and upper teeth and gums on affected side
196
Q

What sort of chemicals can harm the eye?

A
  • Alkalis penetrate ocular tissue and are the most harmful eg ammonia, lye in drain cleaner, caustic potash, Mg(OH)2 from sparklers, lime in wet plaster, cement and mortar
  • Acids usually produce more superficial damage but concentrated acids may produce effects where you can’t distinguish from alkali damage
197
Q

How might an eye look after chemical trauma is sustained?

A
  • Lids- burned or oedematous
  • Conjunctiva- epithelial loss, chemosis, haemorrhage
198
Q

How to treat chemical trauma to the eye?

A
  • Treat as an emergency even before vision testing
  • Immediate management-
    • Measure the pH of both eyes- normal pH is 7.3-7.6
    • Instil topical anaesthetic and eyelid speculum and irrigate the eye copiously with saline or Ringer’s lactate solution delivered via IV tubing for at least 30mins
    • Pull down lower lid and double evert the upper lid to irrigate the fornices and remove any matter
      • Rarely this requires a GA
    • Continue irrigating until normal pH
  • Subsequent treatment
    • Review after 1-2 days for ischaemia
    • Debride clearly necrotic corneal and conjunctival epithelium
    • Use preservative free eye drops
    • Surgical interventions may be required
199
Q

Is orbital cellulitis an emergency?

A
  • YES
  • THREAT TO LIFE if it spreads to intracranial space
200
Q

Possible sources of infection for orbital cellulitis?

Of these which is the most common source of infection?

Also what is the most common causative microorganism of orbital cellulitis?

A
  • Paranasal sinuses (most common)
  • Oropharynx including teeth
  • Foreign bodies
  • Skin
  • Trauma including iatrogenic
  • Haematogenous spread
  • Bacteria most common cause- strep or staph
201
Q

Describe the difference between periorbital and orbital cellulitis?

Which one has more severe complications?

A
  • Periorbital = pre-septal cellulitis
    • Within eyelid tissue, superficial to orbital septum
    • Secondary to superficial infections eg from bites, wounds
    • Ocular function- eye movements and vision- unaffected
  • Orbital = post-septal cellulitis
    • Infection within the orbit, posterior or deep to orbital septum
    • Sight-threatening
    • Potential route
    • Features- proptosis/ exophthalmos, reduced or painful eye movements, reduced visual acuity
    • There’s potential for infection to spread intracranially because orbital veins drain to cavernous sinus, pterygoid venous plexus and facial veins- resulting in cavernous sinus thrombosis or meningitis
202
Q

Clinical features of orbital cellulitis?

A
  • Pre-septal cellulitis- fever, peri-ocular pain, swelling, ptosis, tenderness, redness
  • Orbital cellulitis- similar to the above however with chemosis, conjunctival infection, reduced eye movements, +/- diplopia, vision loss, RAPD (relative afferent pupil defect), proptosis
203
Q

Suggest 4 key Qs to ask in a hx when suspecting orbital cellulitis?

A
  • Sinus disease
  • Trauma
  • Skin infection, dental infection
  • Immunocompromise, diabetes
204
Q

What to do when examining orbital cellulitis pt?

A
  • Mark the skin inflammation to determine if it’s improving or worsening later on
205
Q

How to manage pre-septal cellulitis?

A
  • Mild: amoxicillin/ clavulanate or flucloxacillin. 10 days
    • Then every 2-7 days till improvement
  • Severe: admit for ceftriazone IV daily in divided doses until responding then treat as mild disease
206
Q

How to manage orbital cellulitis?

A
  • Admit into hospital
  • Monitor orbital and visual functions 2-3 times daily
  • Give ceftriaxone IV plus flucloxacillin IV qds
  • Consider vancomycin, clindamycin, teicoplanin or other abx for MRSA
  • In pts >10 years old or those with chronic sinonasal disease, add metronidazole
  • Consider IV treatment 3-5 days, provided condition improves
  • Request CT, blood glucose, FBC, blood culture
  • Refer sinus disease to ENT
  • Repeat CT to exclude abscess if any deterioration occurs
  • Urgent neurosurgical referral if there is neurological deterioration or intracranial abscess
  • Change to oral abx when there is definite improvement
  • Review every 2-5 days after discharge until complete resolution
207
Q

Describe the use of vital dyes to aid diagnosis of ocular disease?

A
  • Dyes are used for diagnostic purposes-
    • In clinic
    • During procedures/ surgeries
  • Preservative free dyes are not recommended- due to contamination of pseudomonas aeruginosa
  • Eg fluorescein- orange, water soluble dye, emits a yellow green light. Some uses include:
    • To look at the cornea- eg see corneal ulcers or abrasions
    • Goldmann tonometer (intraocular pressure)
    • FFA
208
Q

Describe the use of mydriatics in diagnosis of ocular disease?

A
  • Antimuscarinics dilate the pupil & paralyse the ciliary muscle
  • Short acting weak mydriatics eg tropicamide (action lasts up to 6hrs)- facilitate examination of the fundus of the eye
209
Q

What is optic coherence tomography?

A
  • A non-invasive diagnostic test which uses a low powered laser to scan and image the inner layers of the retina
  • OCT can help diagnose various eye diseases- ARMD, macular oedema, glaucoma
210
Q

What is fundus fluorescein angiography used for?

A
  • FFA is a technique for examining the circulation of the retina and choroid using a fluorescent dye and a specialised camera
  • Dilation drops may be required to enlarge pupils
211
Q

How are visual fields assessed? Including imaging techniques

A
  • Sit directly opposite the pt, ~1metre
  • Ask pt to cover 1 eye with their hand, you mirror them and cover the opposite eye on yourself
  • Ask pt to focus on your nose and not move their head or eyes. You also focus on their face
  • Ask pt is any part of your face missing or distorted- this screens for central visual field loss or distortion
  • Position the hatpin at an equal distance between you ant the patient
  • Assess the pt’s peripheral visual field, start from the periphery and slowly move towards the centre, asking the pt to report when they first see it- if you see it and the pt cannot, then they have a reduced visual field
212
Q

What are some types of visual field defects?

A
  • Bitemporal hemianopia
    • Central tunnel vision
    • Typically due to optic chiasm compression by tumour (eg pituitary adenoma)
  • Homonymous field defects
    • Affect the same side of the visual field in each eye
    • Commonly due to stroke, tumour, abscess
    • Half the vision is affected
  • Scrotoma
    • Area of absent or reduced vision surrounded by areas of normal vision
    • Many possible aetiologies- eg demyelinating disease (MS) and diabetic maculopathy
  • Monocular vision loss
    • Total vision loss in 1 eye secondary to optic nerve pathology (eg anterior ischaemic optic neuropathy) or ocular diseases (eg central retinal artery occlusion, total retinal detachment)
213
Q

What is optical biometry & when is it used?

A
  • A highly accurate non-invasive automated method for measuring anatomical characteristics of the eye
  • Accurate measurements are required for determining the correct power of an intraocular lens before it is implanted during cataract surgery
  • The corneal curvature, axial length, focimetry (or refraction) are all measured
214
Q

What is optical biometry & when is it used?

A
  • A highly accurate non-invasive automated method for measuring anatomical characteristics of the eye
  • Accurate measurements are required for determining the correct power of an intraocular lens before it is implanted during cataract surgery
  • The corneal curvature, axial length, focimetry (or refraction) are all measured
215
Q

Briefly describe what happens in cataract surgery?

A
  • Removing the cloudy lens & replacing with the artificial one
  • Mostly local anaesthetic
  • Corneal incision
216
Q

Briefly, what is a retinal laser used for?

A
  • Treatment of proliferative diabetic retinopathy
  • Treatment of diabetic macular oedema
  • Branch and central retinal vein occlusion
  • ARMD
217
Q

When are intravitreal injections required? What are some risks of intravitreal injections?

A
  • Gold standard for treatment of many retinal diseases, such as-
    • Neovascular AMD
    • Diabetic macular oedema/ non proliferative or proliferative diabetic retinopathy
    • Retinal vein occlusions
    • Uveitis
  • Risks-
    • Pain/ FB sensation
    • Bleeding- subconjunctival, vitreous haemorrhage
    • Retinal tear/ detachment
    • Cataract- from inadvertently hitting the lens
    • Infection- endophthalmitis
    • Uveitis
218
Q

Describe the visual requirements for driving? (visual acuity and visual field)

A
  • You must be able to read with glasses or contact lenses if necessary, a car number plate made after 1st September 2001 from 20 metres
  • You also must have a visual acuity of at least 0.5 (6/12) measured on Snellen scale (with glasses or contact lenses if needed) using both eyes together or one eye only if you only have one eye
  • You must have adequate field of vision- horizontal field of vision of at least 120 degrees
219
Q

What are low visual aids?

A
  • Items such as reading stands, anti-glare spectacles and task lights help to aid function during more visual tasks
220
Q

Briefly, what Is Charles Bonnet syndrome?

A
  • Characterised by persistent or recurrent complex hallucinations (usually visual or auditory), occurring in clear consciousness
  • Generally this is against a background of visual impairment
  • Insight is usually preserved
  • Risk factors- advanced age, peripheral visual impairment, social isolation, sensory deprivation, early cognitive impairment
221
Q

What are some causes of monocular diplopia?

A
  • Abnormal refraction
  • Abnormal cornea
  • Abnormal lens
  • Abnormal iris
  • Normal examination- not diplopia
222
Q

What are some causes of monocular diplopia?

A
  • Intermittent/ variable
    • Myasthenia gravis
    • Intranuclear ophthalmoplegia
    • GCA
  • Persistent:
    • Neurogenic- cranial nerve palsies
    • Myogenic- congenital (rare) or acquired (thyroid)
223
Q

What are causes of a red eye + reduced vision?

A
  • Normal IOP:
    • Corneal abrasion
    • Keratitis
    • Endophthalmitis
  • Increased IOP:
    • Acute glaucoma
    • Anterior uveitis
224
Q

What is myasthenia gravis?

A
  • Autoimmune disease, characterised by weakness and fatigability of skeletal muscle
  • Antibodies against post-synaptic Ach receptors (AChR-Ab), bind to ACh at the neuromuscular junction, preventing binding of Ach and subsequent depolarisation needed for muscular contraction
  • Any age, but has bimodal distribution- peaks about 20 years and 60 years
  • Commoner in females
225
Q

Describe the ocular and systemic features of myasthenia gravis?

A
  • Ocular: variable diplopia/ ptosis/ ophthalmoplegia- weakness to eyelids extraocular muscles- usually worsening towards evening/ with exercise
    • Pupillary sparing- no evidence of abnormally small or large pupils and react to light normally
  • Systemic: fatigable weakness of limbs, speech, chewing, swallowing, breathing
226
Q

What are the investigations and treatment for myasthenia gravis?

A
  • Ix-
    • Serum antibodies: anti-Ach receptor is present in >90% of pts with generalised MG, BUT only 50% of ocular myasthenia
    • MuSK antibodies- found in a small % of pts
    • Electromyography
    • Thymus scanning- CT/ MRI
  • Mx-
    • Anticholinesterases- pyridostigmine
    • Prednisolone
    • Immunosuppression- azathioprine
    • Thymoma is an indication for thymectomy
227
Q

What are the key things to ask in an ophthalmology history?

A
  • Sudden or gradual?
  • Painful or painless?
  • Transient or persistent?
  • One or both eyes?
  • Blurred vision- centrally or centrally + peripherally
  • Associated distortion or double vision?
  • Symptoms there all the time?
228
Q

How are the pupils assessed?

A
  • Inspect pupils- size and symmetry
  • Light reaction- observe constriction in the illuminated pupil (direct response) and the constriction of the shielded pupil (consensual pupil)
  • Swinging light test- pen torch beam passed rapidly from one eye to the other, the normal response is sustained constriction of both pupils
    • If one pupil dilates upon illumination there is a relative afferent pupillary defect in that eye
    • Simple test of optic nerve function
  • Accommodation reflex- ask pt to look at distant object and then at an object close to his face, both pupils should constrict and dilate again when distant gaze is resumed
229
Q

How do you assess eye movements?

A
  • H-test- ask pt to keep head still, and with both eyes open, follow your finger (you make a H shape)
    • Pause when they look laterally to look for nystagmus (cerebellar pathology)
    • If there is a complex ophthalmoplegia, ask them to look straight up while counting down from 20 (fatigability – myasthenia gravis)
230
Q

How is colour vision assessed?

A
  • Istihara plates- colour perception test for red-green colour deficiencies
231
Q

How are visual fields assessed?

A
  • Ask pt to cover 1 eye with their palm and cover your eye on the same side too
  • Ask them to stay focussed on your open eye
  • Select a white visual fields pin and bring it in from the periphery, keeping it mid-distance from you and the pt
  • Ask them to tell you when they can see it
  • Move in a diagonal direction into each of the 4 quadrants
232
Q

What sort of visual field abnormalities may be identified?

A
  • Monocular field loss = intra-ocular pathology or ipsilateral optic nerve lesion
  • Bitemporal hemianopia = optic chiasm compression
  • Left/ right homonymous hemianopia = contralateral optic tract/ radiation lesion, or occipital cortex if macular sparing if present
233
Q

How to examine for visual inattention?

A
  • While the pt keeps both eyes open and focussed on you, hold out your hands in each of their outer visual fields, ask them to point to the hand which you are opening or closing
    • Inattention to one side = contralateral parietal lesion
234
Q

What is visual acuity?

A
  • A measure of the ability of the eye to distinguish shapes and the details of objects at a given distance
  • One eye is assessed at a time
235
Q

How is visual acuity assessed in adults?

A
  • Snellen chart
    • Sit pt 6m away from chart
    • Ask pt to cover 1 eye and read lowest line
    • Record the line
      • eg 6/6 (= 20/20). If they read the 6/6 line but get 2 letters incorrect, record as 6/6 (-2)
    • Use a pinhole to see if vision improves- if it does, then this suggests a refractive error
    • If pt unable to read the top line of the Snellen chart at 6m even with pinhole, reduce distance to 3m (record as 3/denominator), then 1m (record as 1/denominator), assess if they can count the number of fingers you’re holding up (record as CF- counting fingers), assess if they can see gross hand movements (record as HM- hand movements), assess if they can detect light from a pen torch shone into each eye (record as PL- perception or light- or NPL- no perception of light)
  • (Assess near vision- fine print reading in a book or newspaper (with 1 eye closed))
236
Q

How is visual acuity assessed in children?

A
  • Preferential looking test- based upon the turning of the head or eyes towards the pattern rather than a uniform field
  • Children with better vision can see finer grating and turn toward it
237
Q

At what level of vision do most people undergo surgery for their cataracts?

A
  • The threshold is 6/12 in the worst eye
  • However, NICE states that measurement of visual acuity can fail to detect vision problems that may benefit from cataract surgery such as glare and loss of colour vision
  • The decision should include consideration of a pt’s quality of life and symptoms such as difficulty with reading, night driving, work or home activities, glare and loss of contrast, despite appropriate optical correction
238
Q

How would you consent someone for cataract surgery?

A
  • Explain the benefit and complications
  • Benefits-
    • Improved quality of vision
    • Improved fundal view to monitor or treat posterior segment disease
    • Reduced spectacle dependence (a reading correction is usually required)
  • Complications-
    • Most complications are infrequent, and most can be treated
    • Risk of blindness is 1 in 1000, with the risk of losing the eye as a result of surgery is 1 in 10,000
    • Immediate: endophthalmitis (bacterial or fungal infection of the intra-ocular fluid)- intravitreal abx required, can lead to loss of vision and the eye
    • Delayed: retinal detachment, macular degeneration, posterior capsule opacification
239
Q

What is the commonest cause of registration as blind in the UK?

A
  • Age related macular degeneration
240
Q

Can drusen ever be normal?

A
  • Small hard drusen- less than 63 micrometres, can be normal
  • When there is 63 micrometres or more, there is risk of progression to AMD
241
Q

Describe the changes that occur in wet ARMD?

A
  • Neovascular (exudative) AMD- new blood vessels in choroid, easily bleed, leak, resulting in distortion & scarring of retina
242
Q

What 2 investigations can be used to investigate a diagnosis of age-related macular degeneration?

A
  • A diagnosis of dry AMD is made using slit-lamp fundus examination alone
  • Optical coherence tomography (OCT) for wet AMD
  • If OCT doesn’t exclude neovascular disease, then fundus fluorescein angiography (FFA) is used
243
Q

What is the most common systemic association of paediatric uveitis?

A
  • Juvenile idiopathic arthritis
  • It is defined as inflammation of 1 or more joints of unknown aetiology, that begins before the age of 16 and persists for at least 6 weeks
244
Q

What long-term ophthalmological complications may develop as a result of uveitis?

A
  • Higher incidence of vision threatening complications eg cataract, macular oedema, glaucoma- which may cause irreversible visual loss
245
Q

What is the tear film?

A
  • Tears wet the ocular surface and a tear film is regenerated every time blinking occurs
  • Tears provide a smooth ocular surface for light rays to be refracted uniformly and to provide lubrication
  • The tear film has 3 components
    • Surface lipid layer- secreted by meibomian (tarsal) glands- prevents evaporation of the underlying aqueous component
    • Middle aqueous layer- secreted by lacrimal gland and accessory lacrimal glands
    • Inner mucus layer- secreted by goblet cells of the conjunctiva and the epithelial cell surface
  • Prevents the ocular surface from drying out
246
Q

What symptoms would a patient experience with dry eyes and what are the mechanisms of a dry eye?

A
  • Grittiness and discomfort of the eye
  • Deficiency of tarsal glands eg obstruction to their openings will lead to increased evaporation of tears
  • Reduced production of tears- consider systemic disease- Sjogren’s or drugs- antihistamines
247
Q

Describe the chambers of the eye

A
  • 2 chambers- filled with aqueous humour
    • Anterior chamber- the space between the cornea and iris
    • Posterior chamber- the space between the iris and the ciliary body and the lens
  • Vitreous chamber- The cavity behind the lens is filled by vitreous humour (transparent, jelly-like substance), supports the lens & holds the retina in place
248
Q

What are the functions of the cornea?

A
  • Maintaining transparency à clear window through which we can see
  • Ocular protection (including corneal reflex)
  • Retraction of incoming light (+ overlying tear film)
249
Q

What are the 5 layers of the cornea?

A
  • Epithelium
    • Non-keratinised stratified squamous layer, 5-7 cells layers thick
    • When damaged, cells from the basal layer must migrate to here to cover the wound
    • Regeneration of the corneal epithelium takes around 3-14 days
  • Bowman’s membrane
    • Acellular layer
    • Separates the overlying epithelium from the underlying stroma
    • Trauma below this level will result in scarring of the cornea because only the epithelium is regenerated
  • Stroma
    • Connective tissue layer, forms the bulk of the cornea
    • Predominantly type 1 collagen fibres w/ keratinocytes (fibroblasts) between the collagen fibres
    • Corneal oedema: visible lines in the stroma (striae)
  • Descemet’s membrane
    • Separates the corneal stroma anteriorly from the endothelium posteriorly
    • Acts as a basement membrane to the endothelium
  • Endothelium
    • Main function is to maintain hydration in order for the cornea to remain transparent
    • A layer of simple squamous endothelium that does not regenerate
    • Cellular loss may be accelerated by trauma, surgery and exposure to UV radiation
250
Q

Describe why the following symptoms occur in keratitis: pain, red eye, reduction in visual acuity

A
  • Loss of epithelium exposes free nerve endings causing severe pain
  • The inflammation also leads to increased circumcorneal vascularity and a red eye
  • Corneal surface and tear film is disrupted along with possible corneal oedema causing a resultant reduction in visual acuity
251
Q

What might an examination of the eye of a patient with keratitis reveal?

A
  • White deposit in the cornea (corneal infiltrate)
  • Collection of pus behind the cornea in the anterior chamber (hypopyon)
  • Looking at the anterior chamber more closely may reveal cells and flare which are leucocytes and protein derived from the leaky iris blood vessels responding to the infection
252
Q

Describe the autonomic control of intraocular pressure?

A
  • Via adrenergic receptors
  • Alpha 2 receptors- stimulation reduces the IOP by reducing aqueous production and may increase uveo-sacral outflow
  • Beta 2 receptors- stimulation increases IOP by increasing aqueous production
253
Q

What are some causes of cataract in younger patients?

A
  • Trauma- direct injury to lens (penetrating), electric shock/ lightening, blunt trauma
  • Drugs- steroids (systemic and topical), amiodarone, phenothiazines
  • Systemic disease- diabetes mellitus, myotonic dystrophy, neurofibromatosis type 2
254
Q

What are the 3 morphological types of cataract?

A
  1. Subscapular
    1. Directly under the lens capsule, granular or plaque-like appearance
    2. Near vision is affected more than distance as opacity at nodal point of eye
  2. Nuclear
    1. Involving lens nucleus. Yellowish-brown due to deposition of urochrome pigment
    2. Pt becomes myopic due to increase refractive index of lens, colours appear less well saturated and more yellow/ brown
  3. Cortical
    1. Cortex of lens, radial spokes in the periphery
    2. Give risk to astigmatic changes, pts troubled more in dark when pupil dilates and exposes more of cataract
255
Q

Describe the functions of the iris?

A
  • The iris controls the amount of light entering the eye by varying the size of the aperture in the middle of the iris- the pupil
  • In conditions of low light/ sympathetic activation- the pupil dilates = mydriasis, allows more light into the eye
    • Dilator pupillae muscle allows the pupil to dilate
  • Bright light- pupil constricts = miosis
    • Sphincter pupillae muscle allows the pupil to constrict
    • Stimulated via parasympathetic activation, the fibres of which are carried in the inferior division of the 3rd nerve
256
Q

List some of the important functions of the choroid?

A
  • Allows nerves and vessels to reach the anterior eye by passing through the choroid
  • Removes waste product from the outer retina
  • Supplies essential nutrients to the outer half of the retina
  • Absorbs any light passing through the retina, thus preventing it from reflecting back and interfering with vision
257
Q

How is the choroid attached to the retina?

A
  • Basement membrane called Bruch’s membrane
258
Q

Describe the blood supply to the retina?

A
  • Highly metabolically active hence requires a good blood supply to meet demands
  • Inner 2/3 supplied by central retinal artery
  • Outer 1/3 supplied by the choroidal blood supply
  • The central retinal artery divides into 4 and each supplies a segment of the retina
    • These arteries are end arteries
259
Q

Describe the pupillary reflex?

A
  • Afferent pupillary fibres travel with the rest of the retinal fibres but leave the visual pathway just before the lateral geniculate nucleus (LGN)
  • After leaving the visual pathway they synapse with the pretectal nucleus
  • From here fibres travel to both Edinger-Westphall nuclei (3rd nerve nucleus)
  • This is important as it demonstrates why stimulation of pupillary afferent fibres in 1 eye causes a bilateral motor pupillary response
  • Efferent fibres leave the 3rd nerve nucleus, travel to the ciliary ganglion (via the inferior division of the oculomotor nerve) and finally to the sphincter pupillae à pupillary constriction
260
Q

List the steps of an eye examination?

A
  • Check visual acuity
  • Check visual fields with the confrontation test
  • Observe both eyes- look for discharge, ptosis, lid lesions
  • Examine corneal reflexes and carry out a cover test to detect any misalignment, check the eye movements in all positions of gaze
  • Examine the pupils and look for any opacity within the normally black pupil, look for irregularity or distortion of the pupil that could be caused by damage to pupil or adhesions between the iris and the lens, check for a relative afferent pupillary defect
  • Examine each eye with pen torch looking for generalised or circumcorneal redness, corneal opacity such as a foreign body or abscess, any fluid level of blood or pus in the anterior chamber, and any distortion or irregularity of the pupil
  • Use a direct ophthalmoscope to examine the red reflex in each eye to check opacities in the ocular media such as those caused by corneal oedema or other opacity, cataract, or vitreous haemorrhage
  • Examine the optic disc, macula and retina with the direct ophthalmoscope, taking care to document both normal and abnormal findings (this is easier when the pupils are dilated)
261
Q

What are the uses of fluorescein drops?

A
  • Diagnostically to highlight defects in the corneal epithelium
  • Diagnostically to assess tear drainage in children with congenital nasolacrimal duct obstruction
  • Can be used as an investigation when measuring the intraocular pressure (tonometry)
  • Can be administered combined with a local anaesthetic as a 0.25% solution with oxybuprocaine HCl or proxymetacaine HCl
262
Q

What is RAPD caused by?

A
  • It is caused by a lesion anterior to the optic chiasm ie optic nerve or retina
    • Retina: detachment
    • Optic nerve: optic neuritis (MS)
263
Q

What is Relative Afferent Pupillary Defect?

A
  • A condition where pupils respond differently to light stimuli shone in one eye at a time due to unilateral or asymmetrical disease of the retina or optic nerve
  • It is found by the swinging light test
264
Q

Describe how partial damage to the right optic nerve can cause RAPD?

A
  • Stimulation of R afferent pupillary fibres causes bilateral pupillary constriction- if the R optic nerve is severed there’s no afferent conduction so there’s no direct or consensual response from shining a light in the right eye
  • When light is shone into the left eye a consensual pupillary response would still occur because the efferent system is still intact and can still be innervated by afferent fibres from the L eye innervating both R and L 3rd nerve nuclei
  • If the R optic nerve was damaged only partly, there would be a sluggish pupillary response when light is shone into the R eye and a normal response when light is shone into the L. This can be exaggerated by swinging light from the R eye to the L and back again- the R eye will dilate (rather than constrict)
265
Q

What are some causes of vision loss that lasts a few seconds?

A
  • Unilateral:
    • GCA
    • Optic disc swelling- infection, inflammation
    • Impending central retinal vein occlusion
  • Bilateral:
    • Disc swelling due to idiopathic intracranial hypertension (visual obscurations)
266
Q

What are some causes of vision loss that lasts a few minutes?

A
  • Unilateral:
    • Amaurosis fugax
    • GCA
  • Bilateral:
    • Vertebrobasilar artery insufficiency
  • Migraine- lasts up to 1 hr
267
Q

State some causes of painful sudden/ recent loss of vision

A
  • Abnormal cornea- keratitis
  • Abnormal disc- optic neuritis
  • Abnormal uvea- anterior uveitis
  • Normal fundus- retrobulbar optic neuritis
268
Q

What is papilloedema?

A
  • Optic disc swelling caused by raised ICP
  • Almost always bilateral
269
Q

What features are observed during fundoscopy that suggest papilloedema?

A
  • Venous engorgement (usually first sign)
  • Loss of venous pulsation- although normal pts may not have a pulsation
  • Blurring of the optic disc margin
  • Elevation of optic disc
  • Loss of optic cup
  • Paton’s lines: concentric/ radial retinal lines cascading from the optic disc
270
Q

What causes papilloedema?

A
  • SoL: neoplastic, vascular
  • Malignant hypertension
  • Idiopathic intracranial hypertension
  • Hydrocephalus
  • Hypercapnia
  • Rarer causes- hypoparathyroidism, hypocalcaemia, vitamin A toxicity
271
Q

How to manage papilloedema?

A
  • Urgent CT head or MRI
  • Referral to neurosurgeons
  • If CT head doesn’t show SoL or anything to increase ICP, can consider LP
272
Q

Causes of tunnel vision?

A
  • Papilloedema
  • Glaucoma
  • Retinitis pigmentosa
  • Choroidoretinitis
  • Optic atrophy